Tax missed Q's round 1

अब Quizwiz के साथ अपने होमवर्क और परीक्षाओं को एस करें!

Reagan purchased stock in XYZ Corporation three years ago for $110,000. Today it is worth $100,000. Reagan exchanges (sells) the stock to a local charity for $10,000. What is Reagan's capital loss on the transaction? A) $0 B) $10,000 C) $100,000 D) $3,000

A) $0 Rationale Answer: A For tax purposes, bargain sale transactions cannot generate capital losses. Answer B, C, & D are incorrect because this is a bargain sale, and no amount of loss is permitted.

Ian, a single taxpayer, received $15,000 of Social Security retirement benefits in the current year. He also received $16,000 of interest income. How much of Ian's Social Security benefits must be included in his gross income? A) $0. B) $7,500. C) $12,750. D) $15,000.

A) $0. Rationale The correct answer is "A". Since the total of Ian's MAGI ($16,000) and one-half of his Social Security benefits (0.50 x $15,000 = $7,500) is less than the base amount ($25,000), none of his Social Security benefits are included in gross income.

In the current year, Susan and Tom had three preschool-aged children who require daycare so that Susan and Tom work. Their total daycare costs for all three children was $6,500. While at daycare, 1/3 of the time is education while the remainder is custodial care. Assuming that Susan and Tom have an AGI of $110,000, what is the dependent care credit amount? A) $1,200 B) $1,300 C) $4,800 D) $6,500

A) $1,200 Rationale The correct answer is "A." Sue and Tom will be able to take a credit of $1,200 ($6,000 x 20%). Although not normally allowable as deductible child care, education expenses for children preschool through kindergarten are qualified expenses. The amount of costs that qualify is the lesser of actual costs or $3,000 for one qualified individual, and $6,000 for two or more qualified individuals (2018). If AGI is great than $43,000 the allowed credit is 20%.

Paul (age 35) and his wife Stacey (age 33) are married with three young children. They both work outside the home. Paul is a corporate executive with Wellstar and Stacey is an executive assistant with a small local company. Paul fully participates in his company's qualified retirement plan by contributing $17,500 of his salary, which is matched 100% up to 3% of compensation. Stacey's employer does not offer a retirement plan. In addition, during the year they had the following items of income and expense: Paul's gross salary: $150,000 Stacey's gross salary: $32,000 Stacey's cash gift to her mother: $5,000 Interest from a joint savings account: $100 Federal income taxes withheld from paychecks: $30,000 State income taxes withheld from paychecks: $12,000 Charitable contributions made: $3,400 Rent paid for apartment: $24,000 Contribution to Paul's traditional IRA: $5,500 Contribution to Stacey's traditional IRA: $5,500 What is Paul and Stacey's adjusted gross income? A) $159,100 B) $164,600 C) $176,600 D) $182,000

A) $159,100 Rationale The correct answer is "A." 150,000 Paul's gross salary +32,000 Stacey's gross salary 182,000 + 100 Interest 182,100 -17,500 His 401K 164,600 -5,500 Her IRAs

Kenny has been a night watchman at Burgundy Company for 10 years. During the current year, he received the following benefits from Burgundy Company: Salary = $15,000; Hospitalization premiums = $3,600; Required lodging on Burgundy's premises as a condition of Kenny's employment = $2,400; Reward for preventing a break-in = $1,000. What amount is includible in Kenny's adjusted gross income for the current year? A) $16,000 B) $17,400 C) $18,400 D) $22,000

A) $16,000 Rationale The correct answer is "A." $16,000 ($15,000 salary + $1,000 reward). Hospitalization premiums paid by the employer and any other benefit required by the employer for the employer's benefit are not included in the employee's AGI income.

When reviewing a client's income tax return from the prior year you notice that they had adjusted gross income of $175,000 and paid federal income tax of $31,500. Assuming that the client's income for this year will closely approximate that of last year, what is the minimum amount to pay in estimates to meet safe harbor? A) $28,350 B) $31,500 C) $34,650 D) $37,800

A) $28,350 Rationale The correct answer is "A." Your primary choices are (A) = 90% of current year, (B) = 100% of prior year, and (C) = 110% of prior year. Since the client's AGI from last year is greater than $150,000 the safe harbor is 90% of current year tax or 110% of prior year tax. Since the client's income "closely approximates that of last year" we can utilize the 90% of current year amount. If the client's income varies widely then we would use 110% of prior year.

In which of the following situations, if any, may the individual NOT be deemed a dependent of the taxpayer: A) A cousin who does not live with the taxpayer. B) A former brother-in-law who does not live with the taxpayer. The taxpayer is divorced. C) A nephew who does not live with the taxpayer. D) A legally adopted child who does not live with taxpayer.

A) A cousin who does not live with the taxpayer. Rationale The correct answer is "A." All other parties either satisfy the relationship or member of the household test. IRS Publication 501: Relatives who don't have to live with you. A person related to you in any of the following ways doesn't have to live with you all year as a member of your household to meet this test. Your child, stepchild, foster child, or a descendant of any of them (for example, your grandchild). (A legally adopted child is considered your child.) Your brother, sister, half brother, half sister, stepbrother, or stepsister. Your father, mother, grandparent, or other direct ancestor, but not foster parent. Your stepfather or stepmother. A son or daughter of your brother or sister. A son or daughter of your half brother or half sister. A brother or sister of your father or mother. Your son-in-law, daughter-in-law, father-in-law, mother-in-law, brother-in-law, or sister-in-law. Any of these relationships that were established by marriage aren't ended by death or divorce.

Abby and Brock are divorced. They have one daughter, Caroline, who spends exactly six months of the year living with her mother and exactly six months of the year living with her father. Abby has an AGI of $200,000 and Brock has an AGI of $150,000. Caroline is a qualifying child of both Abby and Brock. Who can claim Caroline as a dependent? A) Abby. B) Brock. C) Both Abby and Brock. D) Neither Abby nor Brock.

A) Abby. Rationale The correct answer is "A". In this case, both parents are eligible to claim Caroline as a dependent because she is a qualifying child for both of them. However, an individual cannot be claimed as a dependent by more than one person. Because Caroline lives with each parent for the same amount of time, the parent with the higher AGI is allowed to claim the dependency exemption. In this case, Abby has a higher AGI than Brock, so she is allowed to claim a dependency exemption for Caroline.

The constructive receipt doctrine: A) Applies to a secular trust used for deferred compensation. B) Does NOT apply to cash basis taxpayers. C) Is used to distinguish unearned income from earned income. D) Means that a taxpayer cannot plan transactions to defer the recognition of income.

A) Applies to a secular trust used for deferred compensation. Rationale The correct answer is "A." The cash method of accounting recognizes income when received. Receipt may be either actual or constructive. Constructive receipt is when the taxpayer has the right to the money although they are not in possession thereof. A secular trust constructively belongs to the beneficiary therefore constructive receipt applies.

Britney owned an office building in Los Angeles that she rented out to several production companies. The building was destroyed by a fire and was a complete loss. Britney received a settlement from her insurance company and would like to reinvest in a new property. Britney wants to make sure that she is eligible for nontaxable exchange treatment. Which of the following is not correct regarding the requirements for nontaxable exchange treatment on Britney's transaction? A) Because Britney rented out the building instead of using the property directly, the replacement property must meet the functional use test. B) Britney must invest the proceeds in a replacement property that has a similar use to the property that was destroyed in the fire. C) Britney must reinvest the insurance proceeds within two years from the end of the year in which she received the insurance proceeds. D) Since Britney received cash as a result of the involuntary conversion, nonrecognition treatment is not mandatory even if she meets all of the requirements.

A) Because Britney rented out the building instead of using the property directly, the replacement property must meet the functional use test. Rationale The correct answer is "A". The replacement property must meet the taxpayer use test, not the functional use test, since Britney did not use the property directly. The taxpayer use test requires replacement property to be used by the taxpayer in an activity which is treated the same for tax purposes in order to qualify for nontaxable exchange treatment. All of the other statements regarding the nontaxable exchange treatment of Britney's transaction are correct.

Cindy owned her home six months prior to moving into it. She had lived in her home for 18 months when her employer required that she move to another state to manage its sales office. Her realized gain from the sale of her home is $149,000. Does Cindy have to report income, and if so, how much? A) Cindy does not have to report income because she has owned and used her home for 18 months and her gain is less than the pro rata exemption. B) Cindy reports a gain of $149,000 because she did not meet the owned and used rule. C) Cindy may exclude up to 75% of her actual gain under the allowable exclusion because the move was job related. D) Cindy may exclude up to 75% of the allowable exclusion but must purchase a replacement home within two years at a cost equal to or exceeding the selling price.

A) Cindy does not have to report income because she has owned and used her home for 18 months and her gain is less than the pro rata exemption. Rationale The correct answer is "A." Section 121 requires that to qualify for the exemption she must have 1) owned and 2) used as principal residence for two years out of the past 5 years. An exception to the rules exists where a taxpayer moved because of employment transfer. In this case, Cindy owned and lived in the home for 18 of the 24 months. Cindy is entitled to 75% of the allowable $250,000 gain exemption or $187,500.

Which of the following statements regarding depreciation is/are correct? I. Real estate used for residential rental purposes is depreciated on a straight-line basis over 27 1/2 years. II. Real estate used for commercial purposes is depreciated on a straight-line basis over 39 years. III. Both the land and the value of the improvements on the land can be depreciated. IV. A mid-year convention is used in the depreciation of real property. A) I and II only. B) I, II and III only. C) I, II, and IV only. D) I, II, III, and IV.

A) I and II only. Rationale The correct answer is "A". Statement "III" is incorrect because only the value of the improvements on the land can be depreciated. The value of the land itself cannot be depreciated. Statement "IV" is incorrect because a mid-month, not a mid-year convention is used in the depreciation of real property.

Howard is 53 years old and has decided to purchase Long-term Care Insurance. Which of the following most accurately describes the tax benefits of premiums paid on a long term care policy? I. The policy must be guaranteed renewable or non-cancelable for the premiums to be deductible. II. Since Howard is less than age 62, only 10% of premiums paid are deductible. III. Premiums paid are deductible but limited based upon age. IV. The long term care insurance deduction is for AGI. A) I and III only. B) III only. C) II and III only. D) I, II, III and IV

A) I and III only. Rationale The correct answer is "A." The IRS provides guidelines for the amount of premiums that are deductible based upon the insured's age. The amount of premiums paid is included in the medical expense deduction for total expenditures exceeding 7.5% of AGI and is from AGI. The policy must be guaranteed renewable or non-cancelable to be qualified.

For the purposes of Earned Income Credit, which loss/losses are disregarded? I. Net losses from estate and trusts. II. Net losses from non-business rents. III. Net capital losses IV. Taxable interest on U.S. government issue securities. A) I, II and III only. B) I, II, III and IV. C) II and III only. D) IV only.

A) I, II and III only. Rationale The correct answer is "A." Tax has already been paid on federally issued securities, while the rest are all disregarded.

Which of the following best describes depreciation cost recovery? A) It is periodic expensing of tangible property, including real and personal property used in business. B) It is periodic expensing of the cost of intangible and tangible assets. C) It is the periodic expensing of natural resources, tangible and intangible assets, as they are being used up. D) It is an expense that fluctuates with the actual taking of the resource from the land.

A) It is periodic expensing of tangible property, including real and personal property used in business. Rationale The correct answer is "A." Cost recovery is a periodic expensing of tangible property, including real and personal property used in business. Amortization is a periodic expensing of the cost of intangible assets. Depletion is the expensing of natural resources as they are being used up. This expense will fluctuate with the actual taking of the resource from the land.

Which of the following miscellaneous itemized deductions is/are not deductible? A) Losses on IRAs. B) Gambling losses to the extent of gambling income. C) Loss on the disposition of an annuity contract. D) Repayments of income.

A) Losses on IRAs. Rationale The correct answer is "A". Of the above options, only losses on IRAs were subject to the 2% floor under the old law and all miscellaneous itemized deductions subject to the 2% AGI limit were eliminated under TCJA. All of the other options are miscellaneous itemized deductions not subject to the 2% floor.

Mike rents his personal residence out each year for 2 weeks during the national golf championship hosted by his neighborhood golf club. The rental income received is $12,000 each year. Mike's rental related expenses are $1,200. How much will Mike recognize as taxable income from the rental activity? A) None B) $10,800 C) $11,954 D) $12,000

A) None Rationale The correct answer is "A." No income is required to be reported when renting a personal residence for less than 15 days. Expenses are also disallowed in this situation.

Sara and Bill have rental property that was rented this year to a family whose primary bread winner lost his job. As a result they had uncollected rent for 2 months before they began the eviction process and 1 additional month before the family was finally evicted. If the rent was $800 per month, how much of a deduction may Sara and Bill claim on their income tax return for the uncollected rent? A) None, uncollected rent is not deductible. B) $800 of uncollected rent for the month it took to evict the renters. C) $1,600 of uncollected rent before action was taken to evict the renters. D) $2,400 of uncollected rent for the entire delinquency.

A) None, uncollected rent is not deductible. Rationale The correct answer is "A." They cannot deduct from ordinary income an amount that was never included in taxable income. Therefore, Sara and Bill will not receive any deduction for the uncollected rents.

Which of the following is a "trade or business" expense? A) QNEC contribution to 401K plans to retain qualification. B) Charitable contributions by a partnership. C) Gifts to contract letting officals to get preferential treatment in contracts. D) Prepaid parking fines paid at a discount to the city to avoid ticketing in high traffic areas.

A) QNEC contribution to 401K plans to retain qualification. Rationale The correct answer is "A." Charitable contributions by a partnership are personal expenses. Fines are not deductible. Gifts (bribes) are not deductible. QNEC, qualified non-elective contributions, to a 401K plan to maintain qualification is an ordinary/necessary business expense.

Which of the following is not a requirement of the individual real estate investor exception to the passive activity loss rules? A) The taxpayer must materially participate in the activity. B) The taxpayer must own at least 10% of the value of the real estate. C) The taxpayer must have an AGI of less than $150,000. D) The taxpayer must actively participate in the activity.

A) The taxpayer must materially participate in the activity. Rationale The correct answer is "A". The taxpayer is not required to materially participate in the activity, but the taxpayer must actively participate in the activity. Material participation requires substantial, continuous involvement in the operation of the activity. Active participation means that the taxpayer participates in making management decisions concerning the property, but is not substantially and continuously involved in the operation of the activity.

Dina loans $24,000 to her daughter Erin and does not charge any interest. Erin has investment income of $1,400 and investment expenses of $300. The applicable federal rate is 5%. How much interest must be imputed on the loan? A) $1,000. B) $1,100. C) $1,200. D) $1,400.

B) $1,100. Rationale The correct answer is "B". Erin has net investment income of $1,100. Therefore, the amount of imputed interest is the lesser of net investment income or interest calculated using the AFR less interest calculated using the stated rate of the loan. Since the stated rate of interest on the loan is 0%, the amount of imputed interest is the lesser of $1,100 or $1,200 ($24,000 x 0.05). Therefore, $1,100 of interest must be imputed on the loan.

This year, Gail had a Section 179 deduction carryover of $7,800 from last year. This year, she elected Section 179 for an asset acquired at a cost of $10,000. Determine Gail's Section 179 deduction for this year. Her net income is $135,000 for the current year. A) $10,000 B) $17,800 C) $13,900 D) $7,800

B) $17,800 Rationale The correct answer is "B." Section 179 carryover to subsequent years will be limited by the business income before the deduction and the year's Section 179 limitation. In 2018, the limit for Section 179 is $1,000,000. Therefore, the entire amount of $17,800 ($7,800 carryover + $10,000 current year's deduction) is allowable.

Elaine incurred $26,000 of margin interest on her $600,000 investment portfolio. Her portfolio income consists of $10,000 in interest, $15,000 in qualified dividends, $3,000 in ordinary dividends, $6,000 in short-term capital gains, and $11,000 in long-term capital gains. How much of the margin interest is deductible on Elaine's tax return assuming no special elections? A) $17,000 B) $19,000 C) $24,000 D) $26,000

B) $19,000 Rationale The correct answer is "B." She can deduct up to her net investment income which = $10,000 + 3,000 + 6,000 = $19,000 without making a special election. If she elected to treat the long-term capital gains and qualified dividends as ordinary income she could deduct it all. However, DO NOT assume that the election is made. The question would have to specify that information or ask "what is the maximum she can deduct?"

For purposes of tax deductibility what is the minimum deductible that a family can have on a high deductible health policy in the current year? A) $1,350 B) $2,700 C) $3,450 D) $6,850

B) $2,700 Rationale The correct answer is "B." HDHP for family is $2,700 for 2018. $1,350 is for a single person.

Veronica borrowed $300,000 to acquire a parcel of land to be held for investment purposes. During the year, she paid interest of $30,000 on the loan. She had AGI of $70,000 for the year. Other items related to Veronica's investments include the following: Investment income = $15,200 Long-term gain on the sale of stock = $6,000 Investment counsel fees = $900 Veronica is unmarried and elected to itemize deductions. She had no miscellaneous deductions other than the investment counsel fees. Determine Veronica's maximum investment interest deduction. A) $30,000 B) $21,200 C) $16,100 D) $15,200

B) $21,200 Rationale The correct answer is "B." The taxpayer's investment interest deduction is limited to the investment income. The investment income is $15,200 plus she can add the capital gains of $6,000 and deduct $21,200. The excess investment interest ($30,000 - $21,200) can be carried over to next year.

Kevin's 12 year old daughter, Angel, has a brokerage account that generates $13,000 of interest income and $2,000 of qualified dividends for the current year. Angel also has earned income of $13,000 from modelling that she is saving for college. How much will be taxed at Angel's tax rate? A) $1,000 B) $3,100 C) $12,900 D) $16,000

B) $3,100 Rationale The correct answer is "B." The calculation is as follows: UI 15,000 - 2,100 = $12,900 taxed at Trust & Estate rate EI 13,000 always taxed at child's rate (if taxed) Total income $28,000 - 12,000 (SD) = $16,000 - 12,900 (taxed @ T/E rate) = $3,100 taxed at Angel's rate

Isaac is a middle school teacher with gross income this year of $35,000. Based on the following, what is Isaac's adjusted gross income? (1) $4,000 qualified education interest expense (2) $2,000 alimony received under a pre-2018 agreement (3) $1,000 contribution to a traditional IRA A) $30,000 B) $31,500 C) $32,000 D) $33,500

B) $31,500 Rationale The answer is B. Isaac's adjusted gross income is his total gross income of $35,000 - $2,500 in qualified education interest expense (the deductible amount is limited to $2,500) - $1,000 contribution to a traditional IRA = $31,500. The alimony is RECEIVED, which is included in determining his gross income of $35,000. If it was paid then there would be a deduction but that's not the case.

Martha gives her niece a machine to use in her business with a fair market value of $4,200 and a basis of $4,400. What is the niece's basis for depreciation (cost recovery)? A) $0; Gift property is not depreciable. B) $4,200 C) $4,300 D) $4,400

B) $4,200 Rationale The correct answer is "B." Your basis for depreciation is the lower of FMV or adjusted basis for depreciation.

Aurora had the following cash inflows during the current taxable year: (1) Wages: $45,000 (2) Loan Proceeds: $2,000 (3) Child Support: $5,000 (4) Stock Sale Proceeds: $3,000 (5) U.S. Government Bond Interest: $1,000 What is her gross income for income tax purposes if her adjusted tax basis in the stock was $2,000? A) $45,000 B) $47,000 C) $49,000 D) $51,000

B) $47,000 Rationale The answer is B. Wages plus stock sale proceeds less basis in the stock plus U.S. government bond interest = $45,000 + $3,000 - $2,000 + $1,000 = $47,000. Loan proceeds are not taxable income and neither is child support.

George failed to pay $5,000 of income tax due with his return, which was timely filed on April 15th. He waits for 2 months after April 15th to pay the tax. How much will his penalties be? A) $25 B) $50 C) $55 D) $75

B) $50 Rationale The correct answer is "B." The amount for failure to pay is point five percent (0.5%) per month = $5,000 X .005 X 2 = $50 Clock starts April 15. April 15 - May 15 May 15 - June 15 Any time after June 15th and before July 15th would be a full month.

Harvey works for Ice Cream Dream, a company that sells commercial ice cream makers. Ice Cream Dream normally has a gross profit percentage of 30%. Harvey's wife loves ice cream, so he decides to buy her a commercial ice cream maker for her birthday. Harvey paid $650 for a machine that would have normally retailed for $1,000. What, if any, amount must be included in Harvey's gross income? A) $0. B) $50. C) $350. D) $400.

B) $50. Rationale The correct answer is "B". Ice Cream Dream's normal gross profit percentage is 30%. Therefore, Harvey must have paid at least $700 for the machine in order to avoid inclusion in his gross income. However, Harvey's discount was 35%. Therefore, Harvey must include 5% of the discount, or $50, in his gross income.

In the current year Harold had a Section 1231 gain of $13,000. In the prior years, Harold had the following Section 1231 transactions: Year Net Section 1231 Transaction 2017 $4, 000 Loss 2016 $2,000 Loss 2015 None 2014 None 2013 None 2012 $8,000 Gain 2011 $2,000 Gain 2010 $2,000 Gain How will Harold's Section 1231 gain be taxed in the current year? A) $12,000 will be taxed as ordinary income. B) $6,000 will be taxed as ordinary income and $7,000 will be taxed as a Section 1231 capital gain. C) $12,000 will be taxed as a Section 1231 capital gain. D) None of the above.

B) $6,000 will be taxed as ordinary income and $7,000 will be taxed as a Section 1231 capital gain. Rationale The correct answer is "B". 5-year Lookback Rule: Harold would have to recognize $6,000 of his 2018 gain as ordinary income since in 2017 and 2016 he had Section 1231 losses. The remaining $7,000 of Harold's Section 1231 gain ($13,000 - $6,000) would be treated as a Section 1231 capital gain.

Last year, Horace paid the following interest: Interest on his home mortgage = $6,200; Interest on a loan to purchase household furniture for his personal residence = $700; Interest on a loan to purchase State of Nebraska general purpose bonds = $1,400. If Horace itemizes his deductions for last year, the amount of deductible interest expense is: A) $6,900 B) $6,200 C) $7,600 D) $8,300

B) $6,200 Rationale The correct answer is "B." The interest on the loan to purchase household furniture is non-deductible consumer interest. The interest on the loan to purchase State of Nebraska bonds is not deductible under Section 265 because he used the loan to purchase tax exempt municipal bonds.

Ginger, age 21 and a full-time student for a degree at State University, qualifies as a dependent on her parents' return. During the summer, she earned $5,500 from a part-time job. Her only other income consisted of $950 interest on a savings account. What is Ginger's taxable income for the current year? A) $0 B) $600 C) $1,050 D) $5,500

B) $600 Rationale The correct answer is "B." The standard deduction for Ginger is the greater of $1,050 or $350 plus earned income but not to exceed the normal standard deduction. Therefore $350 + $5,500 = $5,850 so it is not limited in 2018. The total income is $5,500 + $950 = $6,450. Taxable income is $6,450 - $5,850 = $600.

Alice owns land "A" with an adjusted basis of $250,000, subject to a mortgage of $50,000. On July 1st, Alice exchanges land "A" and its mortgage for $300,000 in cash, a promissory note for $300,000, and property "B" that has a fair market value of $75,000 with Betty. What is the amount realized by Alice? A) $675,000 B) $725,000 C) $925,000 D) $975,000

B) $725,000 Rationale The correct answer is "B." The realized amount not only includes the monies and fair market value of property "B" received (and any indebtedness the buyer has to the seller), but also any liabilities for the seller is relieved. In this case, the seller received $675,000 in cash, property, and notes (buyers indebtedness to the seller) as well as relief from $50,000 in mortgage. The total amount realized is $725,000.

During the current year, Martin, an unmarried taxpayer residing in Montgomery County, collected $200 interest on U.S. government bonds, $600 on Montgomery County school bonds, and $500 interest on a state condemnation award. He also received $60 in dividends on Ford Co. Common stock. His gross taxable income from the above is: A) $1,360 B) $760 C) $560 D) $260

B) $760 Rationale The correct answer is "B." Interest on Montgomery County school bonds is excluded under Section 103. Gross income is $200 + $500 + $60 = $760.

Olive's daughter Polly suffers from a rare illness. During the current year, Olive drove Polly to see a specialist in another state 15 times. Each trip was 300 miles each way and required an overnight stay in a hotel that costs $70 per night. Olive's AGI is $24,000. What is her medical expense deduction for 2018? A) $0 B) $870 C) $360 D) $2,400

B) $870 Rationale The correct answer is "B". Olive may deduct $0.18 (2018) cents per mile for the travel associated with Polly's medical care and may deduct up to $50 per night per person for lodging. Therefore, the total medical expenses are $2,670 which is calculated as follows: [(300 x 2 x 15 x $0.18) + (15 x $70)]. The trip mileage is stated as 300 miles each way, you will need to multiply by 2 and use round trip mileage. However, Olive may only deduct the amount that exceeds 7.5% of her AGI. 7.5% of Olive's AGI is $1,800. Therefore, Olive's medical expenses deduction is $870. Note, the full $70 is allowed because the rule is $50 per person per night.

Giselle became an AMT taxpayer last year. She had to add several items to her regular taxable income in arriving at alternative minimum taxable income. Which of the following items will result in an AMT credit that can be used to offset future regular tax liability? A) $5,000 in property taxes paid on her principal residence if paid in advance. B) A $75,000 difference between the fair market value of stock and the strike price in the incentive stock option used to purchase the stock. C) $4,000 in interest on private activity municipal bonds. D) $4,000 in additional medical expenses.

B) A $75,000 difference between the fair market value of stock and the strike price in the incentive stock option used to purchase the stock. Rationale The correct answer is "B". The inclusion of the difference between the fair market value and exercise price of the stock options will result in a credit that Giselle can use against future regular income tax liability. The other items are adjustments made to her itemized deductions, which result in permanent differences in tax liability as a result of the imposition of the AMT.

Which of the following is a true statement regarding a current Net Operating Loss (NOL)? A) A NOL generally can be carried back two years prior to the loss year. B) A NOL can only offset up to 80% of the current year's income. C) A NOL deduction is available the year the loss occurs. D) An affirmative election must be made to forgo the carryback period.

B) A NOL can only offset up to 80% of the current year's income. Rationale The correct answer is "B." NOL losses currently cannot be carried back but they can be carried forward, (except for select agricultural or insurance filers). However, the NOL can only offset 80% of the current year's income for years after 12/31/17.

Brady is starting a new business where he will be the sole owner. He would like to have limited liability, but he would prefer flow-through taxation because he expects to have losses in the first few years. He is not concerned about incurring self-employment taxes. Which of the following entities would best suit Brady's needs? A) A General or Limited Partnership. B) A Single-member LLC. C) A S corporation. D) A Proprietorship.

B) A Single-member LLC. Rationale The correct answer is "B". Both the proprietorship and the single-member LLC would provide the flow-through taxation that Brady is looking for, but only the LLC would also provide limited liability.

The accrual method of accounting generally must be used to report income earned by: A) A C corporation that made a S election. B) A general partnership with a corporate partner. C) A LLC that has business in most or all states. D) None of the above.

B) A general partnership with a corporate partner. Rationale The correct answer is "B." The key is the question of when the accrual system "Must" be applied. C Corporations must use the accrual method. Therefore, any entity where a C corporation is a partner or owner must use the accrual method. The correct answer is "B" - Partnerships with corporations as owners must use the accrual method. Option "D" is automatically eliminated because S corporations are not required to use the accrual method of accounting.

Scary Berries, Inc. is a C corporation that specializes in carving berries into frightening images. The company's primary profit generation is the sale of berries and they generate $4 million in annual revenue on average. What accounting method may Scary Berries, Inc. utilize for tax purposes? A) Cash basis B) Accrual basis C) Either cash or accrual D) Units of production

B) Accrual basis Rationale The correct answer is "B." Since Scary Berries generates its income from inventory (the berries) they must file under the accrual basis even though their revenues are under the $25 million accrual basis.

Gee and Bea Lee have a very diverse family. Which of the following children does not meet the relationship test for the purpose of Gee and Bea qualifying them as dependent for that child? A) Dan, Bea's 8-year-old son from a prior marriage. B) Fran, Gee's 10-year-old cousin. C) Stan, a 5-year-old foster child placed with Gee and Bea by a state agency. D) Xan, Bea's 12-year-old niece.

B) Fran, Gee's 10-year-old cousin. Rationale The correct answer is "B". Because Fran is from the same generation as Gee (even though she is presumably much younger than Gee), she does not meet the relationship test. A qualifying child must be from a lower generation than the taxpayer. All of the other options do meet the relationship test for being a qualifying child for the purpose of the dependency exemption.

What classifications of property are subject to cost recovery? I. Personalty. II. Personal use property. III. Natural resources. IV. Intangible property. V. Real estate including land. A) II and IV only. B) I, III and IV only. C) V only. D) All of the above.

B) I, III and IV only. Rationale The correct answer is "B." Personal use property is not subject to cost recovery since it is not used for income generating business purposes. Natural resources are subject to depletion and intangibles are subject to amortization. Personalty assets are used in business and are subject to depreciation. Real estate, as in permanent structures on land, are subject to cost recovery, but the land is not.

What is the major advantage of the cash method of accounting? I. Income is counted at the time it is earned. II. Income may be deferred. III. Deductible expenses may be accelerated. IV. Deductible expenses may be used as carry-backs. A) I and II only. B) II and III only. C) I and IV only. D) III and IV only. E) None of the above.

B) II and III only. Rationale The correct answer is "B." Income may be deferred until cash is received and deductible expenses accelerated if paid.

Depreciation is allowed for which of the following business assets: I. Land II. Tangible personalty III. Stock in trade IV. Improvements to land A) IV only. B) II and IV only. C) I, II and III only. D) I, II, III and IV.

B) II and IV only. Rationale The correct answer is "B." There is no depreciation allowed for inventories, stock in trade or land apart from its improvements. Tangible personalty used in a trade or business is section 1245 property and is depreciable.

Which of the following is not a requirement that must be satisfied in order for a legally married taxpayer to use the head of household filing status? I. The taxpayer must file a separate tax return from the spouse. II. The taxpayer must furnish over one-half of the cost of maintaining the household. III. The spouse must not be a member of the household during the last six months of the tax year. IV. The taxpayer must be legally separated from the spouse. A) II only. B) IV only. C) III and IV only. D) I, II and III only.

B) IV only. Rationale The correct answer is "B". The taxpayer is not required to be legally separated from the spouse in order to qualify as an abandoned spouse and use the head of household filing status. All of the other options are requirements for qualifying as an abandoned spouse.

Which of the following is not an exception to the passive activity rules for rental activities? A) If the average period of customer use is seven days or less, the activity could be considered an active trade or business. B) If the average period of customer use is 30 days or less but the taxpayer does not provide significant personal services in concert with the rental activity, the activity may be classified as an active trade or business. C) The activity will be considered an active trade or business if the rental of property is incidental to the non-rental activity of the taxpayer. D) A rental activity that the taxpayer customarily makes available during business hours for nonexclusive use by customers will be classified as the active conduct of a trade or business, provided the taxpayer materially participates.

B) If the average period of customer use is 30 days or less but the taxpayer does not provide significant personal services in concert with the rental activity, the activity may be classified as an active trade or business. Rationale The correct answer is "B". Option "B" is correct because the taxpayer must provide significant personal services in concert with the rental activity and must materially participate in the activity in order to classify the activity as an active trade or business.

Lisa owns the original copy of the Moaning Myrtle, one of the few paintings created by a renowned renaissance artist and inventor. The Moaning Myrtle has been in Lisa's family for years, but she is getting older and moving into a smaller house and none of her children want the painting. When Lisa inherited the Moaning Myrtle from her father's estate, the value was estimated to be $25,000, but an art expert recently appraised the painting at $750,000. Lisa wants the painting to be cared for, so she donates it to a local art museum, which has agreed to display the painting. Assuming that Lisa's AGI is $100,000 this year, which of the following statements is correct? A) Lisa can deduct $60,000 this year and carry forward $690,000 for the next five years. B) Lisa can deduct $30,000 this year and carry forward $720,000 for the next five years. C) Lisa's deduction is limited to her cost basis of $25,000. D) Lisa's charitable deduction carry-forward will be $730,000.

B) Lisa can deduct $30,000 this year and carry forward $720,000 for the next five years. Rationale The correct answer is "B". Because the Moaning Myrtle will be put to a use that is consistent with the museum's tax-exempt purpose, Lisa can use the fair market value of the painting to calculate her charitable deduction. However, her deduction this year is limited to 30% of her AGI. Therefore, Lisa can deduct $30,000 this year and carry forward the remaining $720,000 of her charitable deduction for up to five years.

Frank is considering selling a parcel of raw land located in South Dakota that he owns. If Frank sells the land, he would like to invest the proceeds in another piece of real property and would like to qualify for like-kind exchange treatment. Which of the following assets would not qualify as like-kind property for the sale of raw land? A) Raw land located in Florida. B) Raw land located in Canada. C) New land located in South Dakota. D) An industrial warehouse located in California.

B) Raw land located in Canada. Rationale The correce answer is "B". U.S. real estate and foreign real estate are not like-kind assets for income tax purposes. Therefore, if Frank exchanges his raw land in South Dakota for raw land in Canada, he will not qualify for like-kind exchange treatment.

Reilly owns and operates an accounting practice as a sole proprietorship. For tax reporting, Reilly uses the accrual method of accounting. Last year, Reilly prepared a tax return for a client and billed the client $600. The client did not pay and has recently disappeared. The reminder notices that Reilly's secretary had sent to the client have been returned with no forwarding address. How should this bad debt be treated for income tax purposes? A) No bad debt deduction is permitted. B) Reilly may deduct $600 from his business income. C) Reilly may deduct $600 as a short-term capital loss. D) Reilly may deduct $600 as a long-term capital loss.

B) Reilly may deduct $600 from his business income. Rationale The correct answer is "B". Since Reilly uses the accrual method of accounting, he reported and paid tax on the $600 of income last year when he performed all of the services necessary to collect the income. It is clear that Reilly will not be able to collect the debt, so he can use the specific charge off method and deduct $600 from his business income this year.

In September of this year, Rudolph refinanced his home. Prior to refinancing, his only outstanding debt was the balance due on his original mortgage of $110,000. Rudolph needed some additional money to pay for his child's college education and to take advantage of an investment opportunity, so upon refinancing, Rudolph took out a 30-year mortgage for $250,000. To reduce the interest rate on the mortgage, down to 5%, Rudolph paid $2,500 in points to refinance. Which of the following statements is correct? A) Rudolph can deduct all of the mortgage interest paid on the note. B) Rudolph can deduct the mortgage interest incurred on $110,000. The remaining interest is not deductible. C) Rudolph can only deduct the mortgage interest incurred on $210,000. D) Rudolph can only deduct the mortgage interest paid on $210,000 plus the points paid on refinancing (ratably over the life of the loan).

B) Rudolph can deduct the mortgage interest incurred on $110,000. The remaining interest is not deductible. Rationale The correct answer is "B". $110,000 of the refinanced amount continues to be treated as acquisition indebtedness since that was the previous balance of Rudolph's mortgage. An additional $100,000 will be considered home equity indebtedness but is not deductible based on its use. Under TCJA only qualified residential interest (acquisition indebtedness plus home improvement portions of additional loans) may be deducted and that is capped at $1 million of debt ($750,000 for contracts after 12/15/17).

All of the following statements concerning the AMT as it applies to individual taxpayers are correct EXCEPT: A) Some itemized deductions taken for regular tax purposes must be added back to regular income to determine income under AMT. B) Taxpayers are permitted to take the standard deduction for both regular and AMT tax purposes. C) All adjustments made to itemized deductions when calculating AMT result in a permanent increase in tax. D) Charitable deductions are claimed in the same manner for regular tax and for AMT tax purposes.

B) Taxpayers are permitted to take the standard deduction for both regular and AMT tax purposes. Rationale The correct answer is "B". Taxpayers who do not itemize deductions take the standard deduction for regular tax purposes, but this is added back to alternative minimum taxable income for AMT purposes. All of the other statements are correct.

Ashton, age 29, purchased stock in a real estate investment trust (REIT) with a dividend yield of 9%. He was concerned that the real estate market may have been hurt in the recent market turmoil. However, they kept paying the stated dividend but his 1099-DIV only lists 4% as ordinary dividends. How can this be possible? A) The REIT was paying 5% in qualified dividends. B) The REIT was making nondividend distributions. C) The REIT was selling his shares. D) The REIT already paid tax on the other 5% so Ashton does not have to.

B) The REIT was making nondividend distributions. Rationale The correct answer is "B." Nondividend distributions are a return of capital and are not included for gross income. (A) is incorrect because REITs cannot pay qualified dividends. (C) is incorrect because the REIT cannot sell an owner's shares. (D) is incorrect because the REIT cannot pay the tax for the shareholder.

A client sold an apartment building last year for $100,000, paying a sales commission of $5,000 plus $2,500 closing costs. The building originally cost $80,000 20 years ago. Total straight line depreciation of $40,000 had been taken. The building had a mortgage of $60,000 which was assumed by the buyer. What is the purchaser's cost basis? A) $70,000 B) $92,500 C) $100,000 D) $107,500 E) $160,000

C) $100,000 Rationale The correct answer is "C." The cost basis to a purchaser is the acquisition cost plus any other costs associated with purchasing the property or making it useful for service. The buyer paid $100,000. The question indicates that it was the seller and not the buyer who paid the sales commission and the closing costs. Therefore, the buyer's basis is only the acquisition costs of $100,000. Cash $40,000 plus mortgage $60,000.

Edgar pays alimony under a divorce decree dated 6/1/17 to his former spouse, Frances, in the following amounts: $150,000 in year 1; $40,000 in year 2; and $20,000 in year 3. How much, if any, recaptured alimony must be added to Edgar's gross income in year 3? A) $0. B) $32,500. C) $112,500. D) $150,000.

C) $112,500. Rationale The correct answer is "C". The alimony recapture in year 3 can be calculated by using the formula: R3 = P1 + P2 - 2P3 - $37,500 R3 is the recapture in year 3. P1 is the payment in year 1, P2 is the payment in year 2, and P3 is the payment in year 3. The amount of alimony recapture in year 3 is $112,500 [$150,000 + $40,000 - (2 x $20,000) - $37,500].

Earl entertains one of his clients on January 1, this year. Expenses paid by Earl are as follows: Cab fare = $22; Dinner at club = $190; Tips to waiter = $38; Cover charge at night club = $40; Presuming proper substantiation, Earl's deductions for the night will be... A) $22 B) $114 C) $136 D) $156

C) $136 Rationale The correct answer is "C." The cab fair is deductible as a transportation expense at the full cost. The business meal and tip are subject to a 50% limitation. The entertainment expenses are not allowed deductions. = $22 + [.50 ($190 + $38)] = $22 + $114 = $136

Chelsea had to put more money into her rental property this year. She had the exterior of the rental home painted and the roof replaced at a cost of $12,500 and $18,000, respectively. How much is depreciable? A) $0 B) $12,500 C) $18,000 D) $30,500

C) $18,000 Rationale The correct answer is "C." Painting, inside or out, is considered a repair, which is immediately expensed. The roof replacement is an improvement that substantially prolongs the asset's life, which is capitalized and depreciated over the useful life.

Ford's federal income tax return was due on April 15 of the current year, but Ford did not file his return or pay his taxes until June 30 of the current year. Ford's unpaid tax balance during this period was $400. What is the total penalty that will be imposed on Ford for his failure to file and failure to pay? A) $60. B) $66. C) $210. D) $204.

C) $210. Rationale The correct answer is "C". The failure to file penalty is 5% of the unpaid tax balance for each month or part thereof that the tax return is late (up to 25% of the unpaid tax balance). Therefore, Ford's failure to file penalty is $60 (3 months x $400 x 5%). However, if a tax return is filed more than 60 days late (as it is in Ford's case), the minimum failure to file penalty is the lower of $210 or 100% of the tax due. Therefore, Ford's failure to file penalty is actually $210. Ford is also subject to a failure to pay penalty of 0.5% per month or part thereof. Therefore, Ford's failure to pay penalty is $6 (3 months x $400 x 0.5%). Note that the failure to file penalty is reduced by the failure to pay penalty. Therefore, Ford's total penalty is $210 ($210 failure to file penalty - failure to pay= $204, plus $6 failure to pay penalty).

Peyton has a piece of equipment used in his business. He exchanges it for a like-kind asset owned by Eli. (Peyton and Eli are unrelated). The basis of Peyton's asset is $40,000 and he gives Eli $20,000 cash plus the asset in exchange for Eli's asset, which is worth $36,000. Eli's basis in his original asset is $10,000. What is Peyton's recognized gain or loss? A) $0 B) $4,000 loss C) $24,000 loss D) $6,000 gain

C) $24,000 loss Rationale The correct answer is "C". While losses are not recognized in like kind exchanges; this is not a like kind exchange. Only property is eligible to like kind treatment under the new law. As a result, Peyton has a realized/recognized loss as follows. Value received (new property) $36,000 - Basis $40,000 - cash $20,000 = $24,000 loss. His new basis is the value he "paid" $36,000.

Mackenzie has two apartment units that are occupied by tenants all year long. In December, the tenants in unit 2 paid him in advance for the next January's rent. The regular rent is $1,000 per month for each of the units. How much rental income must Mackenzie include in taxable income this year? A) $12,000 B) $24,000 C) $25,000 D) It depends on which accounting method he uses.

C) $25,000 Rationale The correct answer is "C." According to Publication 17, "Advance rent is any amount you receive before the period that it covers. Include advance rent in your rental income in the year you receive it regardless of the period covered or the method of accounting you use."

Saul was divorced in 1996 and is now single, age 63. He has gross income of $50,000. His bona fide deductible expenses are as follows: Alimony = $8,000; Charitable contributions = $2,000; Contribution to an IRA = $2,000; Net expenses paid on rental property = $5,000; Interest and taxes on personal residence = $7,000; State income tax = $1,200. What is Saul's AGI? A) $28,000 B) $33,000 C) $35,000 D) $40,000

C) $35,000 Rationale The correct answer is "C." Saul's AGI is calculated as follows: Gross income of $50,000 minus deductions for AGI of $15,000 (Alimony of $8,000, IRA of $2,000, and expenses on rental of $5,000) = $35,000. The others are deductions from AGI or below-the-line deductions.

Mel made the following contributions to charity during the past year: Used clothing of the taxpayer and family - Basis = $900 and FMV = $300 Stock in GMC held as an investment for 13 months - Basis = $8,000 and FMV = $7,000 Stock in United Corp. held as an investment for 9 months - Basis = $9,000 and FMV = $10,000 Real estate held as an investment for six years - Basis = $10,000 and FMV = $25,000 The used clothing was donated to the Salvation Army; the other items of property were donated to a Methodist seminary. Disregarding any percentage limitations, Mel's charitable contribution deduction is: A) $43,300 B) $42,900 C) $41,300 D) $26,300

C) $41,300 Rationale The correct answer is "C." A donation of short-term capital assets is recognized as a charitable expense at the lower of the FMV or basis. Donations of appreciated long-term assets are recognized as a charitable expense at the fair market value unless basis is chosen. In this question, the United Corp stock is a short-term investment that was held for less than 12 months, therefore, it will be expended as a charitable contribution at the donor's basis of $9,000. The remainder of the donated assets are expensed for charitable purposes at their fair market value.

Blake is a CFP® professional and prepares tax returns for his clients. He prepared his brother's income tax return for $1,000 and he willfully neglects to include $30,000 of income since his brother did not receive a 1099 for consulting work. Blake is aware that his brother earned the $30,000 but fails to report it since he doesn't believe the IRS will catch the understatement of income. The additional tax on this $30,000 of income would have been $7,500. How much of a penalty may Blake be subject to for the understatement of income? A) None, but his brother will be subject to penalties. B) $3,750 C) $5,000 D) $7,500

C) $5,000 Rationale The answer is C. The preparer penalty for willful or reckless conduct is the greater of $5,000 or 50% of the income derived by the preparer for the return. In this case, he charged his brother $1,000.

Vince, a single individual, is one of the founders and original shareholders of Security Consulting, Inc., a corporate security consulting firm. The company was initially capitalized with $200,000, and Vince was a 50 percent owner. The company was structured as a C corporation and filing requirements and permissible tax elections that could benefit the owners were made at the time the company was created. After several years of successful operations, Security Consulting lost market share to large national firms, and eventually closed down operations. Since it had no assets other than the goodwill of the business, there was nothing left to distribute to the shareholders. Assuming that there were no changes to Vince's ownership interest over the period of his ownership, and that Vince has no capital transactions in the current year, by how much can Vince reduce his adjusted gross income this year due to the company becoming worthless? A) $3,000. B) $50,000. C) $53,000. D) $100,000.

C) $53,000. Rationale The correct answer is "C". Because it was capitalized with less than $1 million and Vince was an original shareholder, the stock is Section 1244 stock in Vince's hands. Vince can deduct up to $50,000 of losses as an ordinary loss in any one tax year and the remaining loss is treated as a capital loss. Therefore, Vince will be able to deduct $50,000 of his loss as a Section 1244 loss against ordinary income and will qualify for an additional $3,000 long-term capital loss deduction. The remaining capital loss of $47,000 will be carried forward to future tax years.

Kal has taxable income this year of $6 million. He purchased $2,534,000 worth of depreciable property this year and is trying to calculate his §179 deduction. What is the correct amount? A) $530,000 B) $496,000 C) $966,000 D) $1,000,000

C) $966,000 Rationale The correct answer is "C." Since he placed into service more assets than allowed under the limitation you must calculate the phase-out. 2,534,000 placed into service less 2,500,000 placed into service limit = 34,000 $1,000,000 - $34,000 = $966,000. Since this is below the net income of the business there are no further limitations.

Ron wants to make a gift to a state university. Which of the following gifts would provide Ron with the largest charitable deduction for the current tax year if his AGI is $100,000 and he has not made any prior gifts? A) A gift of Long-term publicly traded stock valued at $40,000 with a basis of $10,000. B) A gift of current year inventory valued at $30,000 with a basis of $25,000. C) A cash gift of $55,000. D) Real estate held for years valued at $45,000 with a basis of $10,000.

C) A cash gift of $55,000. Rationale Answer: C A cash gift to a public charity such as the state university provides for a deduction of up to 60% of the donors AGI. This cash gift allows Ron to deduct $55,000. Choice A is incorrect because this stock gift is limited to 30% of AGI. This allows Ron to deduct only $30,000 because it is long term gain property. Choice B is incorrect because the gift of inventory only provides Ron with a $25,000 deduction, limited to basis. This is ordinary income property. Choice D is incorrect because the real estate only allows Ron to deduct $30,000 since it is limited to 30% of his AGI because like A, it is long term capital gain property.

With regard to Sections 1245 and 1250, Section 1231 will be applied only when: A) Any depreciable tangible personal property is sold at a profit. B) Any depreciable tangible personal property is sold at a profit above its adjusted (depreciated) basis. C) Any depreciable property is sold at a profit above its original cost. D) Any depreciable property subject to MACRS rules is sold at a profit.

C) Any depreciable property is sold at a profit above its original cost. Rationale The correct answer is "C." Section 1231 gain is capital gain. Section 1231 gain occurs when the sale price exceeds the original purchase price.

Fiona is a highly compensated employee of GreatWorks, Inc. Which of the following fringe benefits would be taxable to Fiona? A) Health insurance provided by GreatWorks to all employees. B) Group term life insurance in the amount of $40,000 paid for by GreatWorks. C) Dependent care assistance for the highly compensated employees of GreatWorks. D) On-premises athletic facilities that may only be used by the managers and vice-presidents of GreatWorks.

C) Dependent care assistance for the highly compensated employees of GreatWorks. Rationale The correct answer is "C". Dependent care assistance can only be excluded from a highly compensated employee's gross income if it is provided on a nondiscriminatory basis. Answer "D" is not correct because access to athletic facilities can be provided on a discriminatory basis without causing inclusion in the employee's gross income.

Which of the following does not illustrate one of the four basic tax planning principles? A) Buying investments that pay qualified dividends rather than nonqualified dividends. B) Taking advantage of employer-provided health insurance. C) Frequent buying and selling of investments in order to take advantage of the short-term capital gains rates. D) Investing in a Roth IRA.

C) Frequent buying and selling of investments in order to take advantage of the short-term capital gains rates. Rationale The correct answer is "C". Answer "C" does not illustrate one of the four basic tax planning principles. Long-term capital gains rates are more advantageous than short-term capital gains rates; therefore, taxpayers are better off holding investments for at least one year rather than buying and selling on a frequent basis.

Ralph is not married and does not have any children. However, Ralph is a very good son and provides more than half of the cost of maintaining a very nice apartment and the necessary expenses for his mother. Which of the following filing statuses should Ralph use and why? A) Single, because Ralph is not married. B) Single, because Ralph does not have any qualifying children. C) Head of Household, because Ralph's mother qualifies as a dependent. D) Head of Household, because Ralph's mother is a qualifying child.

C) Head of Household, because Ralph's mother qualifies as a dependent. Rationale The correct answer is "C". Because Ralph provides more than half of the cost of maintaining his mother's apartment and necessary expenses, he is eligible to use the Head of Household filing status if he qualifies his mother as a dependent. Option "D" is incorrect because Ralph's mother is a qualifying relative, not a qualifying child. Options "A" and "B" are incorrect; since Ralph is eligible to use the Head of Household filing status, he should use that filing status rather than the less advantageous single filing status.

Eloise is an unmarried elderly woman who lives alone in a small apartment. Eloise is only able to provide 6% of her own support. The remainder of her support is provided by the following people: 10% of her support is provided by her oldest son Frank. 22% of her support is provided by her daughter Gertrude. 30% of her support is provided by her son Henry. 32% of her support is provided by her friend Irene. Which of these individuals is eligible to claim Eloise as a dependent? A) Irene can claim Eloise as a dependent because she provides more support than anyone else. B) Frank can claim Eloise as a dependent because he is the oldest son. C) Henry can claim Eloise as a dependent, but only if Gertrude signs an appropriate statement. D) None of these individuals may claim Eloise as a dependent.

C) Henry can claim Eloise as a dependent, but only if Gertrude signs an appropriate statement. Rationale The correct option is "C". Option "A" is not correct; Irene may not claim Eloise as a dependent because she does not meet the relationship test as a qualifying relative. Option "B" is not correct; Frank has not provided more than 10% of Eloise's support, so he is not a qualifying person. Either Gertrude or Henry may claim Eloise as a dependent because they each provided more than 10% of Eloise's support and together they provided more than 50% of her support. In order for one of them to claim Eloise as a dependent, however, the other must sign a statement agreeing not to claim an exemption for Eloise for this year.

Under which of the following circumstances is a trip outside the United States considered to be purely for business? I. The taxpayer does not have control over the timing or arrangements for the trip. II. The trip outside the United States lasts for less than seven days. III. Less than 50 percent of the time spent on the trip was personal. IV. Vacation was not a primary consideration for the trip. A) I only. B) II and III only. C) I, II and IV only. D) I, II, III, and IV

C) I, II and IV only. Rationale The correct answer is "C". A trip outside the United States is considered to be purely for business when less than 25 percent of the time spent on the trip was personal. All of the other statements regarding travel outside the United States are correct.

Which of the following is a tax credit that reduces the tax due on taxable income? I. Qualified dependent credit. II. Child tax credit. III. Earned income credit. IV. Credit for estimated tax payments. A) I, II and III only. B) II and III only. C) I, II, III and IV only. D) III, and IV.

C) I, II, III and IV only. Rationale The correct answer is "C". The "Qualified dependent credit" is new under TCJA and applies to qualified dependents and/or qualifying children 17 and over. It is limited to $500. The "child tax credit" applies to qualifying children under age 17 and was expanded under TCJA to $2,000 per child, with the possibility of up to $1,400 per child being refundable. The "earned income credit" is a credit against the calculated tax, available to those with very low income, predominantly from earnings (wages) and it is a refundable credit. The CFP exam considers the prepayment of tax, through withholding and/or estimated tax payments, as credits as well since they also reduce the balance due.

Donna sells stock in Martin Corporation to her brother David for $1,800. Donna purchased the stock four years ago for $3,000 and the current fair market value of the stock is $1,800. David paid Donna $1,800 for the stock. Which of the following statements is correct regarding the tax consequences of this transaction? A) If David subsequently sells the stock to an unrelated party for $3,500, he will realize a gain of $1,700. B) Donna has a recognized loss of $1,200. C) If David subsequently sells the stock to an unrelated party for $2,200, he will have no gain or loss. D) If David subsequently sells the stock to an unrelated party for $3,500, he will have no gain or loss.

C) If David subsequently sells the stock to an unrelated party for $2,200, he will have no gain or loss. Rationale The correct answer is "C". David will have a double basis in the stock, determined as follows: If the purchaser's sale price is less than FMV between the original basis & FMV greater than the original basis Then the purchaser's basis used is $1,800 (Loss Basis) no gain or loss $3,000 (Gain Basis)

Lucy and Lou are married and normally file a joint return. Under which of the following circumstances are they required to file a tax return? A) If Lucy is 64 and Lou is 66 and their gross income is $25,000. B) If Lucy and Lou are both 35 and have one dependent and their gross income is $23,000. C) If Lucy is 64 and Lou is 64, Lou is blind, and their gross income is $25,000. D) None of the above.

C) If Lucy is 64 and Lou is 64, Lou is blind, and their gross income is $25,000. Rationale The correct option is "C". The MFJ standard deduction is $24,000 and the additional standard deduction for 65 and over is $1,300. In option "A", their total standard deduction is ($24,000 + $1,300) = $25,300. In option "B", their standard deduction is $24,000. In option "C", their standard deduction is $24,000. Lou may benefit from an additional $1,300 additional standard deduction for blindness upon filing. A return must be filed to claim the ASD for blindness.

What is the maximum capital loss that an individual taxpayer can deduct in any one year? A) None. B) No more than $3,000. C) No more than $3,000 in addition to the total capital gain recognized in that year. D) No more than $3,000 for that year plus any other losses carried forward from prior years.

C) No more than $3,000 in addition to the total capital gain recognized in that year. Rationale The correct answer is "C." The maximum capital loss that a taxpayer can apply is up to the extent of any capital gains for that year plus an additional $3,000 against income from other sources. Any excess losses are carried forward and applied as discussed above until used.

Last year, Paris bought a home in Los Angeles. Paris is now considering selling her home and buying a new home, but she is not sure whether she can qualify for a prorated exclusion of the gain on the sale of her Los Angeles home under Section 121 of the IRC. Under which of the following circumstances would Paris not qualify for a prorated exclusion? A) Paris has decided to sell her house because she has accepted a new job in New York City. Her last job was in Los Angeles. B) Paris has decided to sell her house because her personal physician recommended that she move to the desert in Arizona because the smog in Los Angeles was causing her asthma to get worse. C) Paris has decided to sell her house because her dog Tinkerbell has arthritis and can't walk down the stairs in Paris' home. D) Paris has decided to sell her house because she is bothered by excessive noise from a nearby airport.

C) Paris has decided to sell her house because her dog Tinkerbell has arthritis and can't walk down the stairs in Paris' home. Rationale The correct answer is "C". The health of a pet is not considered a change in health that justifies a partial exclusion under Section 121. All of the other options are reasons that would justify a partial exclusion under Section 121.

Which of the following statements is/are NOT representative of the Federal income tax on individuals? A) Federal Income tax rates are progressive. B) The tax base for the application of income tax rates is taxable income. C) Persons who have earned income other than wages are subject to withholdings. D) All of the above are correct.

C) Persons who have earned income other than wages are subject to withholdings. Rationale The correct answer is "C." All persons with taxable income are subject to the "pay-as-you-go" payment procedure but not withholdings. They must make estimated payments or face interest and penalties.

Our tax laws encourage taxpayers to: A) Sell investment assets that have appreciated, but keep investment assets that have declined in value. B) Sell investment assets that have appreciated, as well as those investment assets that have declined in value. C) Sell investment assets that have declined in value, but keep those investment assets that have appreciated. D) Keep those investment assets that have declined in value as well as those investment assets that have appreciated.

C) Sell investment assets that have declined in value, but keep those investment assets that have appreciated. Rationale The correct answer is "C." Selling assets which have declined in value may result in a loss. Selling appreciated assets will most always result in a taxable gain. The desire of the taxpayer is to pay the least amount of tax. Selling declined assets will assure that.

Which of the following statements concerning the Alternative Minimum Tax (AMT) system is correct? A) Deferral items for AMT purposes result in a permanent increase in tax. B) Exclusion items for AMT purposes may be reversed in future tax years. C) There is an unlimited carryforward for AMT credit generated in a year the taxpayer becomes an AMT taxpayer. D) The AMT credit can be carried back for up to two years, provided that use of the credit does not force the taxpayer to become an AMT taxpayer in those years.

C) There is an unlimited carryforward for AMT credit generated in a year the taxpayer becomes an AMT taxpayer. Rationale The correct answer is "C". The AMT credit that is generated from deferral items in the AMT calcuation may be carried forward indefinitely. The AMT credit may not be carried back. Deferral items may be reversed in future years through the use of the AMT credit, while exclusion items result in a permanent increase in tax.

Wilma is married to Herb, who abandoned her five years ago. She has not seen or communicated with him since June of that year. She maintains a household in which she and her two young dependent children live. Which of the following statements about Wilma's filing status in this year is correct? A) Wilma can use the rates for single taxpayers for this year. B) Wilma can file a joint return with Herb for this year. C) Wilma can file as a head of household for this year. D) Any of the above.

C) Wilma can file as a head of household for this year. Rationale The correct answer is "C." Wilma meets the "abandoned spouse" rules. Therefore, she can file as a head of household. Otherwise, her filing status would be married, filing separately. Head of Household required that the taxpayer pay for more than 50% of the upkeep of the home in which the qualifying individuals reside. The qualifying individuals need not be dependents of the taxpayer.

In year 1 Justin earns $700 from delivering papers for a newspaper company and he is treated as self-employed. In year 2 the newspaper company hires him as an employee and pays him $700 as W-2 income with no federal or state income tax withholding. Does Justin have to file a tax return in either year? A) Year 1: Yes Year 2: Yes B) Year 1: No Year 2: Yes C) Year 1: Yes Year 2: No D) Year 1: No Year 2: No

C) Year 1: Yes Year 2: No Rationale The correct answer is "C." The rule is that a taxpayer must file if he has greater than or equal to $400 of net earnings from self-employment. If the taxpayer does not have self-employment income there is no requirements to filing unless your income exceeds the standard deduction and personal exemption ($0 for 2018 - 2025) for that year.

Harry and Wilma are married and file a joint income tax return. On their return, they report $60,000 of adjusted gross income ($38,000 salary earned by Harry and $22,000 salary earned by Wilma). During the year, they paid the following amounts to care for their 4-year old son and 6-year old daughter while they worked: ABC Day Care Center = $3,000 Nanny who also cleans the house and does laundry = $2,000 Mrs. Smith (Harry's mother) = $1,500 Harry and Wilma may claim a credit for child and dependent care expenses of: A) $2,100 B) $1,000 C) $3,000 D) $1,200

D) $1,200 Rationale The correct answer is "D." The child care credit applies to child care expenses not to exceed $3,000 for one child and $6,000 for two or more children. Based on the AGI, the maximum allowable credit is 20% of the total care costs (to a $6,000 maximum.) $6,000 care cost for two children x 20% = $1,200. It is irrelevant that the nanny also cleans. The money paid to the grandmother is also included in the childcare costs.

Frank's automobile, which he uses exclusively in his trade or business, was damaged in an accident. The adjusted basis of the automobile prior to the accident was $8,000. The fair market value of the automobile before the accident was $10,000 and the fair market value of the automobile after the accident is $500. Insurance proceeds of $9,500 were received. What are Frank's income tax consequences of this transaction? A) $0 B) $1,500 loss C) $500 loss D) $1,500 gain

D) $1,500 gain Rationale The correct answer is "D." His gain is $9,500 - $8,000 because he has already taken depreciation down to $8,000, and he received compensation in excess of his depreciable basis.

Eric's daughter Emilie is attending a local university. Listed below are the items that Eric has paid for this year related to Emilie's education. What are Eric's total qualified tuition and related expenses for the purpose of claiming the American Opportunity credit? $12,000 tuition paid directly to the university. $400 student activity fee which is required by the university for enrollment. $600 in textbooks purchased from an off-campus bookstore. $100 lab fee for a required science course. A) $12,000. B) $12,500. C) $12,700. D) $13,100.

D) $13,100. Rationale The correct answer is "D". Item "I" is a qualified tuition and related expense. Item "II" is a qualified tuition and related expense because it is required by the university for enrollment. Item "III" is a qualified tuition and related expense because textbooks can be purchased directly from the school or another off campus source and still qualify as a qualified education expense. Item "IV" is a qualified tuition and related expense because the class is required as part of Emilie's degree program.

Bonnie and Manuel are married. He paid $100,000 for their home five years ago. Its fair market value was $150,000 when Manuel died. What is Bonnie's basis in the home after Manuel's death if the home was held as community property and Manuel left his half to Bonnie? A) $50,000 B) $75,000 C) $125,000 D) $150,000

D) $150,000 Rationale The correct answer is "D." Upon the death of either spouse in a community property state, both halves of community property are stepped to the fair market value regardless of who inherits the decendant's half.

James and Marilyn Herbert are married and own a vacation home on the beach in Florida. Each summer they are able to rent the property for $1,000 per week. This year they rented the property to six different parties and the total rental period was 133 days. James and his family vacationed there in the fall for three weeks. Expenses for the entire year totaled: Mortgage Interest: $13,500 Mortgage Principal: $4,200 Real Estate Taxes: $6,400 Utilities: $2,300 Trash: $300 Management Fees: $3,800 Depreciation: $10,000 What is the Herbert's profit or loss, and its nature, associated with this property for the current year? A) $21,500 loss, not deductible. B) $8,000 loss, deductible. C) $12,350 loss, not deductible. D) $2,414.87 loss, not deductible.

D) $2,414.87 loss, not deductible. Rationale The correct answer is "D." Interest and taxes are accrued daily. $13,500 + $6,400 = $19,900 x (133÷365) = $7,251.23 Other costs are deducted according to use time (133 + 21 = 154) $16,400 x (133 ÷ 154) = $14,163.64 Total Costs $21,414.87 - Total Revenue $19,000 (133 ÷ 7 = 19 x $1,000) = Loss $2,414.87 not deductible due to mixed use.

Bill and Nancy are married and together they have AGI of $80,000. They have no dependents and they file a joint income return. Each pays $1,200 for hospitalization insurance. During the year, they paid the following amounts for medical care: $7,200 in doctor and dentist bills and hospital expenses and $800 for prescribed medicine and drugs. In December, they received an insurance reimbursement of $1,500 for hospitalization. Determine the deduction allowable for medical expenses paid during the year. A) $8,900 B) $2,400 C) $900 D) $2,900

D) $2,900 Rationale The correct answer is "D." Bill and Nancy can claim a medical expense deduction for the current year of $2,900 determined as follows: Insurance Premiums $2,400; Doctor $7,200; Medicine $800; Total $10,400. Reimbursement $ 1,500, Total $8,900. 7.5% of AGI = $6,000. Deduction $2,900.

Wanda is employed as a retail store manager. For the last calendar year, she had a $100,000 AGI and paid $7,600 in medical insurance premiums. During the year, she paid the following other medical expenses: Doctor and hospital bills for Bob and Sara (Wanda's parents) = $12,000 Doctor and dentist bills for Wanda = $6,400 Prescribed medicines for Wanda = $1,600 Non-prescribed insulin for Wanda = $700 Bob and Sara did not qualify as Wanda's dependents because they have income that requires they file a joint return. Wanda's medical insurance policy does not cover them. Wanda filed a claim for $4,200 for her own expenses with her insurance company in December of last year. She received the $4,200 reimbursement this January. What is Wanda's maximum allowable medical expense deduction for last year? A) $100 B) $18,300 C) $20,100 D) $20,800

D) $20,800 Rationale The correct answer is "D." Medical expenses are an itemized deduction subject to a floor of 7.5% above AGI. The question here is whether non-prescribed insulin is a deductible medical expense. The answer is yes, non-prescribed insulin is deductible as a medical expense. The second issue is that the reimbursement was received this year and the question concerns last year's expenses. The total medical expenses are $28,300 less $7,500 ($100,000 AGI x 7.5%) = $20,800. The code allows a person to deduct the medical expenses for individuals who would be dependents except for income. (Very special exception).

Sam's Turbo Repair, Inc. (STR) purchased a new machine for cleaning and retooling the turbo blades on semi-trucks. The machine cost was $30,000, there was 10% sales tax, and $1,000 delivery and setup fee. What is STR's basis in the new machine? A) $30,000 B) $31,000 C) $33,000 D) $34,000

D) $34,000 Rationale The correct answer is "D." The machine cost of $30,000 plus the sales tax of $3,000 plus the $1,000 freight is the basis for depreciation.

Ellen lost a $40,000 deposit in Tan financial institution when Tan became insolvent. If Ellen has a salary of $50,000, short-term capital gains of $38,000, and no other deductions, determine the maximum deduction Ellen may take with respect to the loss in computing this year's taxable income. A) $31,000 B) $34,900 C) $38,000 D) $40,000

D) $40,000 Rationale The correct answer is "D." Ellen would treat the loss as a short-term capital loss rather than as a personal casualty loss. She could use $38,000 of the Short Term Capital Loss (STCL) to offset the $38,000 capital gains. The remaining $2,000 of the STCL can then be deducted against ordinary income since it is within the $3,000 statutory limit.

Abner owned bonds that paid $750 of interest on the first day of January each year. Exactly one-third of the way through the current year, Abner gave the bonds to his brother, Brody. When Brody receives the $750 of interest on the first day of January next year, what amount will be included in Brody's gross income next year? A) $0. B) $250. C) $500. D) $750.

D) $750. Rationale The correct answer is "D". Brody must include $750 in gross income, but is entitled to deduct $250 in accrued interest.

Chuck purchases land for $250,000. He incurs legal fees of $1,000 associated with the purchase. He subsequently incurs additional legal fees of $15,000 having the land rezoned from agricultural to residential. He subdivides the land and installs streets and sewers at a cost of $500,000. What is Chuck's basis for the land and the improvements? A) $250,000 B) $750,000 C) $765,000 D) $766,000

D) $766,000 Rationale The correct answer is "D." All costs of making property ready for use are capitalized. Here, the acquisition cost, plus legal fees to consummate the acquisition, rezoning costs and property improvement for streets and sewers are capitalized and become a part of the property's basis. The total basis is $766,000.

Pat invests $150,000 for a 10% interest in a limited partnership. He receives a K-1 with his loss at $80,000. How much of his loss is suspended? A) $0. B) $8,000. C) $15,000. D) $80,000.

D) $80,000. Rationale The correct answer is "D". To determine whether any of the losses are suspended you must first apply the at-risk rules, then the passive loss rules. The amount at risk is the basis of $150,000. Since the loss is less than the amount at risk, none of the loss is suspended due to the at-risk rules. In applying the passive loss rules, the passive loss is limited to the amount of passive income for the year. Since there is no passive income for the year, none of the loss may be recognized and the $80,000 loss is suspended.

Tikia, Inc. is filing their corporate income tax return for last year with a tax-year end of September 29. What is their tax accounting period? A) Fiscal B) Calendar C) Part-year D) 52-53 week

D) 52-53 week Rationale The correct answer is "D." A fiscal year ends on the last day of a month other than December. A calendar year ends on the last day of December. A partial-year is for a time span less than 1 year. A 52-53 week year ends on a specified day of the week (such as Friday) that occurs in the last week of the last month of the tax year.

For purposes of determining taxable income, which of the following is true? A) A taxpayer who finds a suitcase of money and spends the money is not required to recognize income. B) The taxpayer who collects income from a customer during the year, but the customer has sued for a refund, can defer recognition of the income until the suit has been resolved. C) Embezzlement proceeds are not included in the embezzler's gross income because the embezzler has an obligation to repay the owner. D) A person who rents out their personal residence 10 days during the year is not required to include any income received.

D) A person who rents out their personal residence 10 days during the year is not required to include any income received. Rationale The correct answer is "D." A person who rents their home for less than 15 days is not required to include the income as it is considered personal property, not rental or mixed use. However, no deducations related to the expense of renting out the home are allowed other than taxes and interest associated with the property that would normally be deductible as an itemized deduction.

Carlos recently won a $20,000 prize for outstanding achievement in his field. Carlos has requested that the prize money be paid directly to his favorite charity. Which of the following requirements must be met in order to exclude the prize money from Carlos' gross income? A) The prize must have been awarded in recognition of religious, charitable, scientific, educational, artistic, literary, or civic achievement. B) Carlos must not have applied for the prize. C) Carlos must not be required to render substantial future services in order to receive the prize. D) All of the above.

D) All of the above. Rationale The correct answer is "D". Options "A," "B" and "C" are all requirements that must be met in order to exclude the prize from Carlos' gross income.

Which of the following factors should be considered in determining whether an activity is treated as an appropriate economic unit for the grouping of passive activities? A) The similarities and differences in types of business. B) The extent of common control. C) The extent of common ownership. D) All of the above.

D) All of the above. Rationale The correct answer is "D." Options "A," "B" and "C" are all considerations in determining the appropriate treatment of an economic unit.

Which income is subject to self-employment tax: A) Self-employment income. B) Income from an LLC when acting as a member-manager. C) Distributive share of a general partner's income regardless of the nature of the partnership. D) All of the above.

D) All of the above. Rationale The correct answer is "D." All of the above answers will subject a taxpayer to self-employment tax.

Capital recoveries include: A) The initial outlay for capital improvements. B) Repair and maintenance expenditures. C) Salvage value. D) Amortization of bond premium.

D) Amortization of bond premium. Rationale The correct answer is "D." Capital recovery is the expensing of certain acquisition costs. Bonds purchased at a premium are amortized over their life to expense the premium paid. The theory is that when they mature, their basis will be equal to their face value and not the face plus premium. Bond expenditures are, therefore, a recovery of capital.

What effect does claiming the Section 179 deduction have on an asset's adjusted taxable basis? A) Immediate expense of the total cost and a resulting basis of zero not subject to recapture. B) Immediate expense of the total cost and a resulting basis of zero subject to recapture. C) Partial expense with a reduced basis and not subject to recapture. D) Any amount of expense not to exceed the total cost, subject to income and Section 179 expense limits, reduces the adjusted taxable basis.

D) Any amount of expense not to exceed the total cost, subject to income and Section 179 expense limits, reduces the adjusted taxable basis. Rationale The correct answer is "D." Claiming Section 179 expense immediately reduces the basis of the property by whatever the amount claimed (not to exceed original cost). The amount of deduction is limited in total and further subject to income limitation before the deduction is taken.

What characteristics of an automobile lease might cause the lease to be treated as a purchase for tax purposes? I. Intent of the parties to the transaction. II. Whether any equity results from the arrangement. III. Whether any interest is paid. IV. Whether the fair market value of the car is less than the "lease payment" or option when the option to buy is exercised. A) I and IV only. B) I, III and IV only. C) I and III only. D) Any of the above.

D) Any of the above. Rationale The correct answer is "D." Any sign of ownership, including that of an installment sale will cause a leased vehicle to be treated for tax purposes as a purchased vehicle. Rather than deducting the lease payments, the taxpayer will be required to elect either standard mileage or to take depreciation plus actual expenses.

Which of the following is a deductible loss? A) Losses on the sale of personal use assets. B) Losses on the subsequent sale of property gifted or sold to a related party when its fair market value is less than the original owner's adjusted basis. C) A wash sales transaction. D) Capital losses in excess of $3,000.

D) Capital losses in excess of $3,000. Rationale The correct answer is "D". Up to $3,000 of capital losses may be recognized against forms of income other than capital gains in any one tax year. However, if a taxpayer has capital losses in excess of $3,000, these losses are not disallowed, but are carried over indefinitely to future tax years.

Last year Dan Walker had a rough year in his taxable brokerage account. He sold when he should have bought, he held when he should have sold and he bought just before prices plunged in most instances. Still he managed to eke out $27,000 in short term capital gains, with only $25,000 of short term capital losses. He also had another $2,000 of investment expenses (research reports). All other considerations aside, he had an AGI of $50,000, so what are the implications of Dan's activities? A) Dan may realize a $2,000 short term gain and deduct $2,000 as a miscellaneous itemized deduction. B) Dan may deduct $25,000 as short term losses and deduct $2,000 as expenses. C) Dan may realize a $25,000 passive activity loss and deduct $2,000 as itemized deduction. D) Dan must recognize a $2,000 short term capital gain, with no other deductions.

D) Dan must recognize a $2,000 short term capital gain, with no other deductions. Rationale The correct answer is "D." He gets to net his portfolio losses against his portfolio gains. He will not be able to deduct his expenses.

Which of the following is not a form on which individual taxpayers can report their income, deductions, exemptions, and other information required to calculate their federal tax liability? A) Form 1040. B) Form 1040A. C) Form 1040EZ. D) Form 1040IND.

D) Form 1040IND. Rationale The correct answer is "D". There is not such thing as Form 1040IND. All of the other forms may be used by an individual taxpayer to calculate their federal tax liability.

Which of the following statements concerning hobby activities is correct? A) Any activity which does not generate a profit within three years must be treated for income tax purposes as a hobby activity. B) The IRS must prove that the taxpayer does not have a profit motive to treat an activity as a hobby activity. C) Expenses associated with the hobby activity can offset, without limitation, the income generated from the activity. D) Hobby income is included in gross income above the line, while hobby expenses are not deductible.

D) Hobby income is included in gross income above the line, while hobby expenses are not deductible. Rationale The correct answer is "D". Income generated from a hobby activity is included in gross income, and expenses associated with the hobby are no longer deductible per TCJA for years after 12/31/17. If the activity earns a profit in three out of five years, the IRS has the burden of proof of showing that there is no profit motive, but if there has not been a profit in three out of the last five years, the taxpayer has the burden of proof. There is no bright line test that requires an activity to be treated as a hobby activity based on the income trend of an activity.

Under which of the following circumstances will a taxpayer be subject to an accuracy-related penalty? I. If the taxpayer files an incorrect return and has failed to make a good faith effort to comply with the tax law. II. If the taxpayer understates his tax liability by more than 5 percent of the correct tax liability. III. If the taxpayer makes a substantial understatement associated with an estate or gift tax valuation. A) I only. B) I and II only. C) II and III only. D) I and III only.

D) I and III only. Rationale The correct answer is "D". A taxpayer will be subject to an accuracy-related penalty if he makes a substantial understatement of his tax liability, generally more than 10 percent of the correct tax liability and at least a $5,000 tax deficiency.

Which of the following statements regarding cost recovery periods is/are correct? I. Most cars are 5-year class life assets. II. Most office furniture is a 7-year class life asset. III. Residential rental real estate is depreciated over 27 1/2 years. A) I only. B) I and II only. C) I and III only. D) I, II and III.

D) I, II and III. Rationale The correct answer is "D". All of the above statements regarding recovery periods are correct.

A minority non-employee shareholder in an S corporation: . I. Receives income when the corporation declares and pays a dividend. II. Votes for the Board of Directors at the annual shareholders' meeting. III. Receives a K-1 annually in order to prepare a personal income tax return. IV. Reports on a personal income tax return a pro rata share of the corporate profit or loss. A) I, II and III only. B) I and III only. C) II and IV only. D) I, II, III and IV.

D) I, II, III and IV. Rationale The correct answer is "D." As a shareholder (employee or non-employee), you are entitled to dividends and voting your pro rata share of ownership. The K-1 will provide all shareholders with their share of corporate income or loss.

Ned, a college professor, owns a separate business in which he participates during the current year. He has one employee who works part-time in the business. Which of the following statements is correct? A) If Ned participates for 120 hours and the employee participates for 120 hours during the year, Ned does not qualify as a material participant. B) If Ned participates for 95 hours and the employee participates for 5 hours during the year, Ned probably does not qualify as a material participant. C) If Ned participates for 500 hours and the part-time employee participates for 520 hours during the year, Ned still qualifies as a material participant. D) If Ned participates for 600 hours and the part-time employee participates for 1,000 hours during the year, Ned nevertheless qualifies as a material participant.

D) If Ned participates for 600 hours and the part-time employee participates for 1,000 hours during the year, Ned nevertheless qualifies as a material participant. Rationale The correct answer is "D." The rules for material participation are: 1. More than 500 hours of participation 2. Taxpayer is the only one who substantially participates 3. Taxpayer spends greater than 100 hours in the tax year and no one else spends more 4. Taxpayer has materially participated in any 5 of the previous 10 years 5. The activity is a personal services activity and the individual has materially participated in any 3 prior years 6. Taxpayer participates 100 or more hours in this activity and total participation in all such activities exceeds 500 hours A is incorrect because he would be a material participant. The rule is > 100 hours and no one spends more. They can spend the same, but not more. (#3) B is incorrect because he is the only one who substantially participates (#2) C is incorrect because he needs to spend more than 500 hours or at least the same as the highest working person to be a material participant. (#1, #3) D is correct because he spent more than 500 hours (#1)

Which of the following is correct? A) Realized gains are always recognized and realized losses are never recognized. B) Realized gains and realized losses on the sale of personal use assets are not recognized. C) Realized gains and realized losses on the sale of personal use assets are recognized. D) In an involuntary conversion, the date of realization, not the payment date, determines the date of recognition.

D) In an involuntary conversion, the date of realization, not the payment date, determines the date of recognition. Rationale The correct answer is "D." Option "A" is incorrect because realized gains may, in some instances, be deferred or excluded such as the gain on sale of a personal residence. Option "B" is incorrect because realized gains on some personal use assets are recognized unless excluded. Option "C" is incorrect because realized losses on personal use assets are not recognized.

Which of the following is not an add-back item for purposes of calculating the Alternative Minimum Tax (AMT)? A) Depreciation of property placed in service after 1986. B) The standard deduction, if taken in lieu of itemized deductions. C) Passive activity losses. D) Installment sales undertaken before March 1, 1986.

D) Installment sales undertaken before March 1, 1986. Rationale The correct answer is "D." Installment sales executed after March 1, 1986 but not before, are add-back items.

Terry and Jim are both involved in operating separate illegal businesses. Terry operates a gambling business and Jim operates a drug running business. Both businesses have gross revenues of $500,000. The businesses incur the following expenses: Terry's employee salaries = $200,000 Terry's bribes to police = $25,000 Terry's rent and utilities = $50,000 Terry's cost of goods sold = $-0- Jim's employee salaries = $200,000 Jim's bribes to police = $25,000 Jim's rent and utilities = $50,000 Jim's cost of goods sold = $125,000 Which of the following statements is correct? A) Terry should report a profit of $225,000. B) Jim should report a profit from his business of $100,000. C) Jim should report a profit from his business of $500,000. D) Jim should report a profit from his business of $375,000.

D) Jim should report a profit from his business of $375,000. Rationale The correct answer is "D." Jim's profit would be as follows: $500,000 (income) - $125,000 (cost of goods sold) = $375,000 of profit. Terry's profit would be as follows: $500,000 (income) - $200,000 (wages) - $50,000 (rental expenses) = $250,000 of profit. Please note that illegal expenses such as bribes to police are neither allowable nor deductible. If the illegal business is drug running then you are limited to only deducting the cost of goods sold.

Mortimer is an avid collector of antiques associated with the funeral industry. The local hospital is running a campaign to redecorate and expand their lobby, and as a show of support, Mortimer donates a 19th century horse-drawn hearse in mint condition to the hospital. He purchased several of these hearses 30 years ago for $300 each, but the current estimated market value of the hearse today is in the range of $30,000. The hospital decides to sell the hearse and dedicate the proceeds to the renovation effort. Mortimer's AGI is $50,000. Which of the following statements concerning Mortimer's charitable deduction is correct? A) Mortimer's deduction for the current tax year will be limited to $25,000. B) Mortimer's deduction for the current tax year will be limited to $15,000. C) At least $5,000 of the deductible amount will have to be carried over to future tax years. D) Mortimer's income tax deduction is $300.

D) Mortimer's income tax deduction is $300. Rationale The correct answer is "D". When tangible personal property donated to a charity will not be used by the charity to carry out its tax-exempt purpose, the deduction available to the donor is limited to the donor's cost basis and will be subject to the 50 percent limitation. Redecorating the lobby is not part of the hospital's tax-exempt purpose. In this case, Mortimer's cost basis is $300 and since 50% of his AGI is $25,000, Mortimer may take his entire charitable deduction this year.

In calculating a net operating loss for an individual, which of the following items would not be added back to negative taxable income? A) Net operating loss deduction B) Long-term capital losses in excess of short-term capital gains. C) Section 1202 exclusions D) None of the above would be added back.

D) None of the above would be added back. Rationale The correct answer is "D." In general, the following items are not allowed when figuring an NOL. · Any deduction for personal exemptions. · Capital losses in excess of capital gains. · The section 1202 exclusion... · Nonbusiness deductions in excess of nonbusiness income. · Net operating loss deduction. · The domestic production activities deduction.

simple java timer Time Elapsed: 22min 47sec4031-RQuestion 19 of 25Income Tax Planning Quiz 8 In calculating a net operating loss for an individual, which of the following items would not be added back to negative taxable income? A) Net operating loss deduction B) Long-term capital losses in excess of short-term capital gains. C) Section 1202 exclusions D) None of the above would be added back. Rationale The correct answer is "D." In general, the following items are not allowed when figuring an NOL. · Any deduction for personal exemptions. · Capital losses in excess of capital gains. · The section 1202 exclusion... · Nonbusiness deductions in excess of nonbusiness income. · Net operating loss deduction. · The domestic production activities deduction.

D) None of the above would be added back. Rationale The correct answer is "D." In general, the following items are not allowed when figuring an NOL. · Any deduction for personal exemptions. · Capital losses in excess of capital gains. · The section 1202 exclusion... · Nonbusiness deductions in excess of nonbusiness income. · Net operating loss deduction. · The domestic production activities deduction.

Which of the following imposed the first constitutional federal income tax? A) Revenue Act of 1861. B) 16th Amendment. C) Revenue Act of 1916. D) None of the above.

D) None of the above. Rationale The correct answer is "D". Answer "A" is incorrect because although the Revenue Act of 1861 did impose a federal income tax, it was later found to be unconstitutional because Congress did not have the power to levy an individual income tax at that time. Answer "B" is incorrect because the 16th Amendment gave Congress the power to impose an individual income tax, but did not itself impose that tax. Answer "C" is incorrect because the Revenue Act of 1916 raised the rates previously imposed under the Revenue Act of 1913. Therefore, answer "D" is correct because the Revenue Act of 1913 imposed the first constitutional income tax.

For tax purposes, a deduction is allowed for the consumption of the cost of an intangible asset through: A) Depletion. B) Depreciation/Cost Recovery. C) Goodwill. D) None of the above.

D) None of the above. Rationale The correct answer is "D." Cost recovery of an intangible asset is allowed through amortization. Cost recovery and depreciation (one in the same) are applied to tangible assets. The costs of natural resources are recovered through depletion.

Which of the following is an itemized deduction? A) Job related moving expenses. B) Tax return preparation fee C) Unreimbursed employee expenses D) Nonprescription insulin.

D) Nonprescription insulin. Rationale The correct answer is "D." Medical expenses including nonprescription insulin are itemized and deductible in excess of 7.5% of AGI. The other choices are no longer deductible under TCJA.

Which of the following statements is correct? A) If an activity is classified as a trade or business, any net loss may be deductible against other income. B) If an activity is classified as a hobby, expenses can only be deducted against the income for AGI (not net losses). C) If an activity is classified as a hobby, the expenses are not deductible. D) Only A and C are correct.

D) Only A and C are correct. Rationale The correct answer is "D." Business losses are deductible against other income presuming the activity is active and the person is a material participant. If the activity is deemed a hobby, the income is taxable, but no deduction is allowed for the related expenses.

Which of the following decreases a taxpayer's at-risk amount? A) Cash and the adjusted basis of property contributed to the activity. B) Amounts borrowed for use in the activity for which the taxpayer is personally liable or has pledged as security property not used in the activity. C) The taxpayer's share of amounts borrowed for use in the activity that is qualified non-recourse financing. D) Passive losses which are used against passive income from another source.

D) Passive losses which are used against passive income from another source. Rationale The correct answer is "D." Options "A," "B" and "C" all increase the at risk amounts.

As a direct result of the rules under TCJA 2017, qualifying dividends will be treated in which manner: A) Qualifying dividends are taxed in the same way as capital gains at an 18% rate for those in the 28% and higher marginal tax bracket. B) Qualifying dividends are taxed at a newly instituted 5% tax rate. C) The taxation on dividends have not been impacted under TCJA. D) Qualifying dividends are taxed at set dollar breakpoints.

D) Qualifying dividends are taxed at set dollar breakpoints. Rationale The correct answer is "D." Under prior law, the capital gain breakpoints were related to the tax breakpoints. Under TCJA the capital gain breakpoints are at set dollar amounts not corresponding to the current tax brackets. See provided tax tables in Helpful Documents within Blackboard.

Which of the following is incorrect? A) All business deductions are classified as deductions FOR AGI. B) Some personal deductions are classified as deductions FROM AGI. C) Some business and some personal deductions are classified as deductions FOR AGI. D) Some business and some personal deductions are classified as deductions FROM AGI.

D) Some business and some personal deductions are classified as deductions FROM AGI. Rationale The correct answer is "D." Deductions occur above the line (for) AGI and below the line (from) AGI. All business deductions are for AGI (above the line). All business expenses (taken by their owners, i.e. sole proprietor or partner) are taken above the line or FOR AGI. A "business deduction" is tied to the business or owners. These are reported on the sole proprietor's schedule C, or a partner's K-1 (which flows onto the Schedule E of Form 1040), for example. Business deductions are all above the line (FOR AGI). "Personal deductions" are not tied to a business. For example, mortgage interest is a personal deduction below the line (FROM AGI). An HSA contribution is a personal deduction above the line (FOR AGI). "Job-related employee expenses" used to be deductible as a miscellaneous itemized deductions subject to the 2% floor, a below the line deduction (FROM AGI). Those are no longer available for tax years 2018-2025.

If personal use property is converted to business use: A) Gain is recognized on the date of conversion to the extent of the excess of the fair market value over the adjusted basis. B) Loss is recognized on the date of conversion to the extent of the excess of the adjusted basis over the fair market value. C) There is no gain or loss from the later sale of the property if the sales price is below the original adjusted basis and over the fair market value at the date of conversion. D) The adjusted basis is the lower of the taxpayer's adjusted basis or the fair market value on the date of conversion.

D) The adjusted basis is the lower of the taxpayer's adjusted basis or the fair market value on the date of conversion. Rationale The correct answer is "D." The basis of personal property converted to business use will be the taxpayer's adjusted basis on that property as of the date of conversion or the FMV if lower. Option "A" is incorrect because no gain on conversion is recognized. Option "B" is incorrect because no loss on conversion is recognized. Option "C" is incorrect because the statement pertains to the basis of gifted property.

Tab owns an apartment building and a DVD rental business. He participates for approximately 600 hours in the apartment building operations and approximately 1,000 hours in the DVD rental activity. Which of the following statements is correct? A) Both the apartment building and the DVD rental businesses are passive activities. B) Neither the apartment building nor the DVD rental business is a passive activity. C) The DVD rental business is a passive activity, but the apartment building is not. D) The apartment building is a passive activity, but the DVD rental business is not.

D) The apartment building is a passive activity, but the DVD rental business is not. Rationale The correct answer is "D." Rental real estate is by definition a passive activity, therefore, the apartment rental business is passive. The DVD rental business is not a passive activity in that DVD rentals are short-term. To be considered as passive activities, other rental activity of goods and equipment must be long-term. According to code, short-term is defined as less than seven days.

The carrybacks and carryforwards associated with the general business credit must be used in a specific order. Which of the following correctly describes that order? A) The business credit carrybacks to the current year; the amount of the current year business credit; and the business credit carryforwards to the current year. B) The business credit carrybacks to the current year; the business credit carryforwards to the current year; and the amount of the current year business credit. C) The amount of the current year business credit; the business credit carrybacks to the current year; and the business credit carryforwards to the current year. D) The business credit carryforwards to the current year; the amount of the current year business credit; and the business credit carrybacks to the current year.

D) The business credit carryforwards to the current year; the amount of the current year business credit; and the business credit carrybacks to the current year. Rationale The correct answer is "D". Answer "D" correctly describes the sequence in which the carrybacks and carryforwards associated with the general business credit must be used.

Which of the following must be capitalized by a business? A) A major tune-up of a truck used in the business. B) The replacement of a windshield of a business truck, which was broken in an accident. C) The repair of a roof damaged by a hurricane. D) The cost of a concrete pad upon which to place a machine.

D) The cost of a concrete pad upon which to place a machine. Rationale The correct answer is "D." Costs to improve, better, or extend the life of an asset are capitalized. Tune-ups are a maintenance activity. Maintenance and repairs (roof and windshield) are period costs deductible in the period in which they were incurred.

Sarah is a 10 percent owner in Canine Connection, LLC, a day-care center for dogs. She is also a 15 percent owner in Little Laughter, LLC, a successful children's clothing store. She does not materially participate in either business. Her at-risk and loss/income for the current year is as follows: Canine Connection-At-risk = $175,000; Loss of $275,000 Little Laughter-At-risk = $25,000; Income of $125,000 She also has wage income of $80,000 and capital gain income of $30,000. Which of the following statements is true? A) The loss suspended because of the at-risk rules is $75,000 and the loss suspended because of the passive activity loss rules is $75,000. B) The loss suspended because of the at-risk rules is $75,000 and the loss suspended because of the passive activity loss rules is $0. C) The loss suspended because of the at-risk rules is $50,000 and the loss suspended because of the passive activity loss rules is $100,000. D) The loss suspended because of the at-risk rules is $100,000 and the loss suspended because of the passive activity loss rules is $50,000.

D) The loss suspended because of the at-risk rules is $100,000 and the loss suspended because of the passive activity loss rules is $50,000. Rationale The correct answer is D. At Risk Passive Loss Suspended Under At Risk Passive Activity Income (Loss) Total Canine Connection, LLC 175 275 <100> <175> <275> Little Laughter, LLC Irrelevant Irrelevant 0 125 125 <100K> <50K> <150K> Loss suspended because of the at risk rules: $275,000 loss - $175,000 at risk = $100,000 suspended. We can net the $125,000 of income against the loss of $275,000, which equals $150,000, minus $100,000 suspended because of the at risk rules, which leaves $50,000 suspended because of the passive activity rules.

Which of the following statements correctly reflects the automatic mileage method (as used to arrive at automobile expenses)? A) The rate includes parking fees and tolls. B) The method does not preclude the later use of MACRS on the same vehicle. C) The election of the automatic mileage method precludes a later change to the actual operating cost method. D) The method can be used on a car that is either purchased or leased by the taxpayer.

D) The method can be used on a car that is either purchased or leased by the taxpayer. Rationale The correct answer is "D." You can still deduct parking fees and tolls, MACRS is not permitted--nor is bonus depreciation such as 179--and you may switch between the standard mileage method and the actual operating cost method. The method does allow use on leased autos but the only exception is "If you want to use the standard mileage rate for a car you lease, you must use it for the entire lease period."

Section 1245 recapture does not apply to business equipment held for 17 months or longer if: A) The property was destroyed by fire and the insurance recovery exceeds the property's adjusted basis. B) The property was sold for a gain but was depreciated using straight-line depreciation rather than MACRS. C) The property was acquired, depreciated, and exchanged for a less valuable asset where the buyer of the asset paid additional cash in the exchange. D) The property was abandoned as worthless.

D) The property was abandoned as worthless. Rationale The correct answer is "D." Option "A" is incorrect because if the insurance proceeds exceed the property's adjusted basis, the excess is considered a sale and any portion of gain attributable to depreciation will be subject to Section 1245 recapture. Option "B" is incorrect because Section 1245 recovery occurs any time a gain results from the reduction of basis due to depreciation. Option "C" is incorrect because Section 1245 applies to gain resulting from a reduced basis due to depreciation. Option "D" is correct. Property sold or abandoned below the basis adjusted by depreciation is not subject to Section 1245 recapture because either not all depreciation was taken or there was more likely a loss rather than a gain. For 1245 recapture to occur there must be a gain over the basis.

During the current tax year, Sam Malone had $10,000 of passive income from a publicly traded limited partnership. He also has a non-publicly traded limited partnership which generated a $10,000 passive loss. How much of the passive loss is deductible by Sam during the current tax year? A) $0 B) $1,000 C) $3,000 D) $10,000

A) $0 Rationale The correct answer is "A." Income from a publicly traded limited partnership may not be offset by any other passive losses.

Under the terms of a divorce agreement dated 1/3/18, Larry is required to pay his wife Joyce $2,100 per month in alimony for a minimum period of 10 years and $300 per month in child support. For a twelve-month period, Larry can deduct from gross income (and Joyce must include in gross income): A) $0 B) $25,200 C) $3,600 D) $28,800

A) $0 Rationale The correct answer is "A." The $300 per month for child support is not deductible by Larry. Child support payments are not deductible to the payor nor includable to the payee. Larry's $300 per month in child support will remain part of his gross income. The $2,100 is not alimony since it could extend beyond her death as the required payment is for a minimum of 10 years.

Hal is an investment counselor who is employed by Richardson Investments. While he attended a conference on the impact of the new tax laws in investment choices, he incurred the following unreimbursed expenses: Airfare = $350; Lodging = $450; Meals = $330; Tuition and fees = $410. Without regard to his AGI, how much can Hal deduct on his return? A) $0 B) $975 C) $1,150 D) $1,375

A) $0 Rationale The correct answer is "A.". Post 12/31/17, TCJA eliminated the deduction for unreimbursed employee business expenses.

Two years ago, Green Corporation, a cash basis taxpayer, sold services to Albert for $25,000. During the prior year, Green collected $10,000 from Albert. In the current year, Green collected $5,000 from Albert in final settlement of the debt. The proper treatment for the bad debt deduction is: A) $0 for the prior year and $0 for the current year. B) $0 for the prior year and $10,000 for the current year. C) $15,000 for the prior year and $0 for the current year. D) $15,000 for the prior year and $5,000 income for the current year.

A) $0 for the prior year and $0 for the current year. Rationale The correct answer is "A." A cash basis taxpayer does not recognize income not received. Since the bad debt was never recognized as income, it cannot be recognized as a loss or a bad debt expense.

Diane purchased a hotel on November 15, five and 1/4 years ago for $5,000,000. Determine the cost recovery deduction for one month. A) $10,683 B) $15,152 C) $21,368 D) $128,200

A) $10,683 Rationale The correct answer is "A." To depreciate real property, the mid-month convention is used. In addition, a hotel is not considered residential real property and is therefore depreciated using straight line depreciation over 39 years 1 ÷ 39 = .02564 $5,000,000 x 2.564% = $128,200 of annual depreciation expense $128,200 ÷ 12 = $10,683 for one month's depreciation in the current year

Last year, Monique took a trip from Boston to Rome. She was away from home for 15 days. She spent five days vacationing and ten days on business for her clothing line (including the two travel days.) Her expenses are as follows: Airfare = $2,100; Lodging (15 days) = $3,150; Meals (15 days) = $2,400; Valet service (cleaning of business suits) = $60. Monique's business travel expenses deduction is: A) $4,360 B) $4,660 C) $5,060 D) $5,860

A) $4,360 Rationale The correct answer is "A." She owns her business and can take a business deduction. If she was an employee of the company it would be zero under the TCJA suspension of itemized deduction subject to the 2% floor. Airfare Deductibility Rules: DOMESTIC If primarily business then deduct all airfare. Prorata meals and lodging. FOREIGN Prorata meals and lodging. Prorata airfare unless (then you can deduct all): < 7 days <25% on personal No control Vacation not a deciding factor Airfare = $2,100 x (10 ÷ 15) = $1,400 Lodging = $3,150 x (10 ÷ 15) = $2,100 Meals = $2,400 x (10 ÷ 15) = $1,600 x 50% (meals are only 50% deductible) = $800 Valet = $60 Total = $4,360

James, a cash basis taxpayer received the following compensation and fringe benefits from his employer in the current year: Salary = $60,000 Bonus for services = $8,000 Premium for disability income protection = $2,000 Premium for group hospitalization insurance = $2,500 The bonus is being paid in January of next year. The wage continuation insurance would pay James three-fourths of his regular salary if he became disabled. The insurance benefits are provided by the employer to all full-time employees. What is James' gross income in the current year? A) $60,000 B) $62,000 C) $62,500 D) $68,000

A) $60,000 Rationale The correct answer is "A." The premiums for disability insurance and group hospitalization insurance are not included in the taxpayers income. Because James is a cash basis taxpayer, income is recognized when received. So the salary is $60,000 in the current year and though he earned his bonus in the current year, he will not recognize it until next year when it is paid.

In the current year, Tom and Tammy Williams, a married couple, file a joint federal income tax return. During the current tax year, the Williams made deductible IRA contributions of $3,000. They are in a 24% marginal income tax bracket. What amount of tax savings is generated by this deduction? A) $720 B) $2,190 C) $3,000 D) $3,846

A) $720 Rationale The correct answer is "A." $3,000 multiplied by the 24% marginal income tax bracket equals $720.

Kelly owns a potato chip manufacturing business. The demand for her gourmet potato chips is on the rise. This year her business had a record taxable income year of $750,000. Kelly has decided to invest in an upgraded potato slicing machine that will slice potatoes four times as fast as her previous machine. The cost of the new potato slicer is $2,650,000 and will be the only depreciable property that Kelly places into service during the current year. What is the amount of her Section 179 deduction for the current year? A) $750,000 B) $850,000 C) $1,000,000 D) $2,650,000

A) $750,000 Rationale The correct answer is "A". Section 179 deductions are limited to $2,500,000 of equipment placed into service. Placing the full 2.5 million would give you 1 million in deductions. Amounts over the 2.5 million will reduce the maximum deduction dollar for dollar. Kelly's Potato Chip manufacturing business' taxable income was $750,000. Kelly cannot deduct more than her taxable income. $2,650,000 - $2,500,000 = $150,000; $1,000,000 - $150,000 = $850,000 limited to the $750,000 of taxable business income.

What is the primary advantage of using the Section 179 Deduction over other cost recovery methods? A) By deducting more currently, total tax liability is reduced and the present value of cash flows is increased. B) The $1,000,000 Section 179 limit allows a businesses to deduct more up front. C) Section 179 reduces the depreciation on most assets to only three years. D) Depreciation applies only to business assets, whereas Section 179 applies to business and personal assets.

A) By deducting more currently, total tax liability is reduced and the present value of cash flows is increased. Rationale The correct answer is "A." Section 179 is an upfront business deduction, now at $1,000,000 (2018) that can be used by businesses to reduce tax liabilities. It's possible to reduce Section 179 deduction to zero, depending how much is placed into service. If too much is placed into service, Section 179 would not have any advantages over other methods of depreciation.

Which is the best source for obtaining a plain language understanding about the current tax law? A) Commerce Clearing House Federal Tax Guide. B) Congressional Tax Committee Reports. C) Treasury Regulations. D) Tax Court Reports.

A) Commerce Clearing House Federal Tax Guide. Rationale The correct answer is "A." Option "A" is correct because Commerce Clearing House (CCH) provides plain language interpretation of tax law. Option "B" is incorrect as the Congressional Committee Reports (sometimes known as the Blue Book) provides congressional reasoning for enacting tax law. This language is often very technical and difficult to understand. Option "D" is incorrect because Tax Court Reports provide rulings of the U.S. Tax Court in the form of case law.

One of the five tests which must be met to qualify as a dependent for a qualifying relative is: A) Gross income of the dependent. B) The ability of the dependent to provide for their own care. C) The age of the dependent. D) Whether the dependent owes income taxes from a previous year having filed single.

A) Gross income of the dependent. Rationale The correct answer is "A." The five dependency tests are: 1) Gross Income Test, 2) Support Test, 3) Not a qualifying child, 4) Citizenship Test (U.S., Canada or Mexico), and 5) Joint Filing Test.

Michael purchased a mutual fund with a $25,000 five years ago. During the four years, $6,000 in dividends and capital gains were reinvested in the fund. Today, the fund is valued at $40,000. If Michael sells shares equal to $25,000, which statement(s) is/are correct? I. The taxable gain can be based on an average cost per share. II. The client can choose which shares to sell, thereby controlling the taxable gain. III. To minimize the taxable gain today, the client would sell shares with the higher cost basis. IV. The client will NOT have a gain as long as he or she sells less than what he or she invested. A) I, II and III only. B) I and III only. C) II and IV only. D) IV only.

A) I, II and III only. Rationale The correct answer is "A." Option I is correct because it is permissable to determine the per share of mutual funds using the average cost per share. Option II is correct because it is permissable to select specific identifiable shares of mutual funds to sell. Option III is correct because the higher cost basis will minimize the gain as long as the taxpayer uses the specific indentification method. Option IV is incorrect because the sales proceeds will be matched against the basis of the shares sold not the original plus adjusted investment.

If an individual who may otherwise qualify as a dependent does not spend funds that he or she has received (i.e., social security, wages), what is the IRS position regarding these unexpended amounts in terms of their application to the support test and their inclusion in being applied to the gross income test? A) Income received but not spent is applicable to the gross income test but not the support test. B) Income received but not spent is not applicable to the gross income test nor to the support test. C) Income received but not spent is applicable to the gross income test and to the support test. D) Income received but not spent is not applicable to the gross income test but is applicable to the support test.

A) Income received but not spent is applicable to the gross income test but not the support test. Rationale The correct answer is "A." Income received but not spent is applicable to the gross income test but not the support test. To claim someone as a dependent you have to provide at least 50% of their support. If the child has income that just goes into savings or spent on miscellaneous fun items instead of being used toward paying their bills (housing, clothing, food, etc.), while you are paying their bills, then you are supporting them. The income counts as gross income for the child, but it is not support unless they use it to support themselves.

Your client Bebe Rebozo is contemplating the exchange of two parcels of investment land for two similar parcels in two separate transactions. Given the following details of the proposed transaction, compute the amount of recognized gain and loss (if any) on both parcels if your client completes the exchanges: Parcel A: Ten acres of land acquired 15 years ago with a current basis of $50,000. In exchange your client will receive eight acres of land (FMV = $80,000) and $20,000 in cash. Parcel B: Twenty acres of land acquired two years ago with a current basis of $100,000. In exchange, your client will receive twelve acres of land (FMV = $75,000) and $10,000 in cash. A) Parcel A Recognized Gain = $20,000; Parcel B Recognized Loss = $0 B) Parcel A Recognized Gain = $20,000; Parcel B Recognized Loss = $10,000 C) Parcel A Recognized Gain = $50,000; Parcel B Recognized Loss = $10,000 D) Parcel A Recognized Gain = $20,000; Parcel B Recognized Loss = $15,000

A) Parcel A Recognized Gain = $20,000; Parcel B Recognized Loss = $0 Rationale The correct answer is "A." This question pertains to like kind exchanges where boot is involved. The rule is that any realized gain will be recognized to the extent of the lesser of realized gain or boot received. In this case, there was a realized gain of $50,000 ($50,000 basis for $100,000 market value). The boot of $20,000 is recognized as gain since it is the lesser of boot or realized gain. Parcel B will have no gain in that there is no realized gain between the basis of the property given up and the fair market value of the property received. There is a realization but it is not recognized. Losses in like kind exchanges are not recognized.

What is the marginal income tax rate? A) The tax rate applied to the last dollar of taxable income earned. B) The lowest tax rate a taxpayer can pay. C) The tax rate on business profit (margin). D) The average tax rate for an individual.

A) The tax rate applied to the last dollar of taxable income earned. Rationale The correct answer is "A." The marginal tax rate is the tax rate applied to the last dollar earned.

Albacore, Inc., an accrual method taxpayer, was incorporated on January 2 this year but did not begin business operations until July 1. Albacore adopted a calendar year tax year and incurred the following expenses during its first tax year: Incorporation fees paid to State: $150 Expenses in connection with issuing and selling stock: $1,800 Legal fees incident to incorporation: $1,650 If Albacore, Inc. makes an appropriate and timely election, the maximum organizational expenditures that it can properly deduct for the current year would be: A) None B) $1,800 C) $3,450 D) $3,600

B) $1,800 Rationale The correct answer is "B." Expenses incurred in connection with issuing and selling stock are not deductible. The rule is the lesser of expenditures or $5,000. Therefore, $1,650 + 150 = $1,800.

Iris, a widow, elected to receive the proceeds of a $50,000 face value life insurance policy on the life of her deceased husband in ten annual installments of $6,800 each beginning in 2009. In the current year, she received $6,800. Iris died in December of this year after collecting the installment payment. What is the amount subject to income tax on her final return? A) $0 B) $1,800 C) $5,000 D) $6,800

B) $1,800 Rationale The correct answer is "B." $50,000 ÷ 10 = $5,000 principal each year. $6,800 - $5,000 = $1,800 of interest. The $1,800 of interest must be included in Iris' gross income. Proceeds from a life insurance policy on the death of an insured are never taxable. Interest received even from installation payments of the proceeds of a life insurance policy are taxable as ordinary income. Because Iris received $1,800 in interest while alive, the interest will be included in the gross income on her final return. She receives no deduction for her remaining unrecovered basis because it is from life insurance. If it had been from an ordinary commercial annuity, she would have a deduction for any nonrecovered basis in the annuity.

In June of this year, Mary's office building was damaged during a sudden storm. The fair market value of the building is $225,000 and her basis is $100,000. The storm caused $30,000 in damage. The insurance company awards Mary $20,000 in a settlement. What is Mary's includible gain or deductible loss? A) $30,000 deductible loss. B) $10,000 deductible loss. C) $20,000 includible gain. D) $0; this is a partial loss and not deductible.

B) $10,000 deductible loss. Rationale The correct answer is "B." Mary's net loss is $10,000 ($30,000 in damage less $20,000 in insurance proceeds) because this was business property.

Alexander Dumas has a salary of $80,000, dividends of $20,000 and limited partnership income of $15,000. This year, he also invested in an equipment-leasing partnership where he is not a material participant. His initial investment included $50,000 cash and a non-recourse note for $60,000. What is the maximum tax deduction Alexander may take on the equipment leasing investment this year? A) $0 B) $15,000 C) $35,000 D) $50,000

B) $15,000 Rationale The correct answer is "B." This is a passive activity. His deduction is equal to his limited partnership income from other sources to the extent he is at risk. The maximum investment deduction may not exceed the cumulative investment income. The $60,000 non-recourse note is irrelevant to answering this question. This question also assumes that the investment occurred in the current tax year.

Earl went from Portland, Oregon to New York City on business. After six days of business meetings promoting his new winery in hopes of expanding distribution, he took four days of vacation to go sightseeing. Earl's expenses for the trip are as follows: Airfare = $1,200 Lodging (10 days x $90) = $900 Meals (10 days x $100) = $1,000 Airport limo = $60 Earl's business deduction is: A) $2,400 B) $2,100 C) $1,620 D) $1,896

B) $2,100 Rationale The correct answer is "B." The expenses associated with a trip that are for non-business purposes are not deductible from income. Remember that meals are only 50% deductible. Accordingly, the answer is calculated as follows: $1,200 (airfare) + $60 (limo) + $540 (hotel - 6 days at 90/day - the remaining 4 days are personal and not deductible) + $300 (meals - 6 days at 100/day x 50% meal allowance) = $2,100. The airfare (to and from) was transportation for business that would not change whether it was personal or not, so it is not pro-rated but rather fully deductible. $1,200 + $60 + $540 + $300 = $2,100. Airfare Deductibility Rules: Domestic: If primarily business then deduct all airfare. Prorata meals and lodging. Foreign: Prorata meals and lodging. Prorata airfare unless (then you can deduct all): < 7 days <25% on personal No control; vacation not a deciding factor

Your client is the sole shareholder of a closely-held corporation. In the current year, the IRS has deemed the operation to be a Personal Holding Company (PHC) because of involvement in a number of investments other than the stated business purpose. It has cited the business as having undistributed holding company income. What are the possible implications of this decision for your client? A) A penalty of 35% can be imposed on the value of the assets of the PHC because of potential fraud. B) A penalty tax of 20% can be imposed on the undistributed PHC income. C) A penalty tax of 35% can be imposed on the undistributed PHC income. D) Because the corporation is a PHC, there is no penalty tax, but the PHC will be required to pay tax at the 20% corporate level.

B) A penalty tax of 20% can be imposed on the undistributed PHC income. Rationale The correct answer is "B." If the business is deemed to be a PHC by the IRS, then a penalty tax of 20% can be imposed on the undistributed personal holding company income.

Section 1245 recapture applies when: A) A gain on sale of real property occurs due to the depreciated basis. B) All or a portion of gain on tangible personal business property resulted from depreciation taken. C) Tangible personal business property is sold regardless of whether there is a gain. D) A gain on any class of property occurs due to cost reduction.

B) All or a portion of gain on tangible personal business property resulted from depreciation taken. Rationale The correct answer is "B." Section 1245 recapture is treated as ordinary income for the gain realized resulting from depreciation taken that was greater than economic reality.

Donna Bella, whose AGI is $250,000, also has passive income of $125,000 and passive losses of $150,000. She uses $125,000 of her passive losses to offset the passive income and the rest is subject to suspension. She has come to you today to find out which of the following ideas is the best possibility for reducing her current tax liability. A) An investment in an oil by-product and natural gas limited partnership generating losses. B) An investment in an activity producing credits. C) An investment in rental real estate as an active participant designed to generate losses. D) An investment in limited partnership historic district rehab project producing both credits and passive losses.

B) An investment in an activity producing credits. Rationale The correct answer is "B." In this case, more losses are not required at all. Rather, credits only are needed to improve the client's tax position.

The best source for gathering information about the intent of recent changes in the tax law might be: A) RIA Federal Tax Coordinator. B) Congressional Committee Reports. C) Treasury Regulations. D) Tax Court Cases.

B) Congressional Committee Reports. Rationale The correct answer is "B." Option "A" is incorrect because RIA provides plain language interpretation of tax law. Option "C" is incorrect because it is the highest level of tax regulations, but does not indicate the intent of Congress in enacting tax law. Option "D" is incorrect because it provides ruling of the U.S. Tax Court in the form of case law. Option "B" is correct because the Congressional Committee Reports (sometimes known as the Blue Book) provides congressional reasoning for enacting tax law.

When do the recapture rules for Section 179 apply: I. When the asset is sold before it would have been fully depreciated. II. When the business use drops below 50%. III. When the Section 179 deduction taken in one year exceeds the allowable maximum. IV. When there is sufficient income in one tax year to support the deduction taken. V. When the deduction causes the tax liability to become negative. A) III and IV only. B) I and II only. C) V only. D) All of the above.

B) I and II only. Rationale The correct answer is "B." Section 179 recapture rules apply when the business use of an asset drops below 50% for a given year or when the asset is disposed of before it would have been fully depreciated.

Which of the following credits are fully refundable? I. The Earned Income credit. II. The American Opportunity credit. III. Lifetime Learning credit. IV. Child Tax credit. V. Adoption credit. A) I, IV and V only. B) I only. C) I and V only. D) None of the choices.

B) I only. Rationale The correct answer is "B." The Earned Income credit is refundable; able to create a negative tax liability. The American Opportunity credit and Child tax credit may be partially refundable.

To which of the following entities does the term "pass-through" entity apply? I. LLC. II. C corporation. III. Not for Profit Corporation. IV. S corporation. V. Partnership. A) I, III and IV only. B) I, IV and V only. C) I, III, IV and V only. D) II and III only.

B) I, IV and V only. Rationale The correct answer is "B." The S corporation, LLC and Partnership are considered "pass through" entities. "Pass through" means that the entity is not taxed separately from its owners, but passes its profits and losses through to the owners in their pro rata share of ownership.

Casualty losses are: I. Reduced by $100 per event if the asset loss was held for personal use. II. Generally, not deductible if they are personal. III. Reimbursements are subject to inclusion in the taxpayers AGI. IV. Reimbursements reduce the loss amount. A) I and II only. B) II and IV only. C) I, III and IV only. D) I and IV only.

B) II and IV only. Rationale The correct answer is "B." Personal casualty losses are not deductible, unless the loss was as a result of a federally declared disaster. For business losses, reimbursements are not included in AGI but rather reduce the loss.

Alisha Syrmos, a CFP® Professional and fee-only financial planner, has assisted Bob Martin, a self-employed physician in tax and investment planning during the year. Identify the schedule(s) on which Alisha's fee may be deductible by Bob on his federal income tax return. I. Schedule A - itemized deductions. II. Schedule C - profit or loss from business. III. Schedule D - capital gains and losses. A) I only. B) II only. C) I and II only. D) I, II and III.

B) II only. Rationale The correct answer is "B." Tax planning fees may no longer be deducted against a taxpayer's itemized deduction, Schedule A. However, because the taxpayer is self-employed, the portion of services related to the business and not personal may be taken as a deductible expense on the taxpayer's Schedule C.

Bill and Renee are married and in community property but living apart and filing separate federal income tax returns. Each earned a salary of $25,000 and Renee received $5,000 in interest on money she inherited from her deceased mother after her marriage to Bill. Which of the following is correct? A) In some states, Bill's gross income is $55,000. B) In some states, Renee's gross income is $27,500. C) In all community property states, Bill's gross income is $25,000. D) In all community property states, Renee's gross income is $30,000.

B) In some states, Renee's gross income is $27,500. Rationale The correct answer is "B." The inherited property may be considered separate property, and therefore, in some community property states the income earned from it is separate. In others, it is community.

Jay owns an office building that burned down. The basis of the building was $125,000. The insurance policy paid Jay $195,000. What can Jay do that will defer the recognition of any gain? A) Jay must use the money to buy similar property. B) Jay must replace the property with similar property of equal or greater value within two years from December 31st of the year of realization. C) Jay must replace the property with any kind of business property within two years from the date of the fire. D) Jay must replace the property with similar property that costs at least as much as what the policy paid within 3 years of the fire..

B) Jay must replace the property with similar property of equal or greater value within two years from December 31st of the year of realization. Rationale The correct answer is "B." Insurance proceeds which exceed the current basis of destroyed property will not be taxable if the taxpayer replaces that property with similar property within a two-year period from the end of the year in which realization resumed if a natural disaster (fire) or three years from the end of the year in which realization occurred in the event of a government taking (emminent domain).

Suppose a company agrees to pay an executive employee $150,000 per year plus 20% of profits for the coming year. The company has a banner year and the employee is paid compensation of $750,000 as a result. Would this likely be deemed unreasonable, and therefore, non-deductible? A) Yes, because the amount is unreasonably high whether it was a good year or not for the company. B) No, because the reasonableness of salaries is tested according to the circumstances under which the agreement was made. C) Yes, because the reasonableness of salaries is tested according to the circumstances when the compensation is actually paid. D) No, because the employee will have to pay tax on the compensation anyway.

B) No, because the reasonableness of salaries is tested according to the circumstances under which the agreement was made. Rationale The correct answer is "B." While $750,000 in compensation may appear unusually large, in this instance it can be considered reasonable because the employee took some risk in accepting this arrangement. The compensation also was deemed appropriate at the time at which it was made. Therefore, the $750,000 that may otherwise be deemed unreasonable is (in this case) within the limits of reasonableness.

Nathan sustained serious facial damage in a snow skiing accident. To restore his physical appearance, he had restorative surgery. At the same time, he also had surgery to remove wrinkles from under his eyes caused by age. Which of these procedures can Nathan claim as a deductibe medical expense? A) Both of the procedures can be claimed as deductible medical expenses because he had them done at the same time. B) Only the restorative surgery can be claimed as medical expenses. C) Only the wrinkle removal surgery can be claimed as medical expenses. D) None of these can be claimed as medical expenses because they are both cosmetic.

B) Only the restorative surgery can be claimed as medical expenses. Rationale The correct answer is "B." Reconstructive surgery to correct disfigurement is a deductible medical expense subject to 7.5% AGI. Elective cosmetic surgery is not deductible.

In the past, the IRS was known to cite parties who were owner-employers of businesses structured as C-corporations as receiving too much salary compensation. Amounts were disallowed and recharacterized as dividends. This practice is expected to reduce during the period while JGTRRA is in force. Why might one expect this? A) Reduction of top marginal brackets means less money in income tax revenue. B) Reduction of dividend tax rates means that dividends, which were once taxed at ordinary tax rates, may net less tax to the government than compensation. C) Reduction of marginal tax rates and dividend rates, which are still not a deductible expense to the corporation, means less corporate income tax revenue. D) Reduction of top marginal brackets indicates an IRS softening of the hard-line position once taken in seeking out revenue.

B) Reduction of dividend tax rates means that dividends, which were once taxed at ordinary tax rates, may net less tax to the government than compensation. Rationale The correct answer is "B." At one time, the higher salaries an owner/employer took, the less profit for the corporation. However, with the reduction of the dividend rate to 15% for most taxpayers (20% for high income taxpayers), the IRS has more incentive to pursue revenue collection from salaries of these individuals.

Under the First in First Out (FIFO) inventory system: A) The first good purchased is the first good sold. B) The cost of goods sold is based on the costs of the first goods purchased. C) FIFO reduces the probability of scrap or obsolescence. D) All of the above.

B) The cost of goods sold is based on the costs of the first goods purchased. Rationale The correct answer is "B." The FIFO method is concerned with movement of costs through inventory, not goods. The cost of the first units purchased are the first costs to be transferred to cost of goods sold when the goods are sold.

One of the five tests which must be met to qualify as a dependent is: A) The age of the dependent. B) The dependent is either a member of the taxpayers household or meets the criteria for family relationship. C) The taxpayer is a U.S. citizen. D) All of the above.

B) The dependent is either a member of the taxpayers household or meets the criteria for family relationship. Rationale The correct answer is "B." The five dependency tests are: 1) Gross Income Test, 2) Support Test, 3) Member of Household or Family Member Test, 4) Citizenship Test (U.S., Canada or Mexico), and 5) Joint Filing Test.

Julian purchased 100 shares of Home Depot, a domestic corporation, common stock on July 7th this year. The ex-dividend date for their quarterly dividend is July 12th. Julian sells the Home Depot stock on August 15th of this year. If Home Depot paid a dividend of $10 on Julian's 100 shares, what are the tax consequences to Julian if Julian is in the 25% tax bracket? A) No tax liability since the qualified dividend rate is 0% for individuals in the 25% bracket. B) $1.50 of tax liability since the qualified dividend rate is 15% for individuals in the 25% bracket. C) $2.50 of tax liability since the ordinary dividend rate is the taxpayer's marginal rate. D) Not enough information to answer the question.

C) $2.50 of tax liability since the ordinary dividend rate is the taxpayer's marginal rate. Rationale Home Depot dividends will qualify for qualified dividend treatment if the individual meets the requisite holding period, which is more than 60 days in the 121 days surrounding the ex-dividend date. Since he only held the stock for 39 days, he does not meet the holding period. Therefore, the tax rate applied against the dividend is his ordinary income tax rate of 25%.

Carl Borden is a contractor who has just purchased a tractor for use in his business. Carl paid $25,000 plus $1,250 in sales tax for the tractor. The local municipality also imposes an annual personal property tax of $500. The tractor has an expected useful life of 5 years. What is Carl's basis in the tractor for depreciation purposes? A) $25,000 B) $25,500 C) $26,250 D) $26,750

C) $26,250 Rationale The correct answer is "C." The basis of depreciable property begins with the acquisition costs plus any additional expense necessary to acquire or making such property ready for use. Here, sales tax is a required expense by law and is therefore added to the acquisition cost resulting in the depreciable basis. $25,000 (cost) + $1,250 (sales tax) = $26,250.

Marsha has the following income and losses for the current year: I. ($1,000) loss from a 30% interest in Laminate Partnership in which she does NOT materially participate. II. ($1,500) loss from a 2% limited partnership interest in Venture, a limited partnership. III. ($3,000) loss from a 12% interest in an S corporation in which she manages one of the departments. IV. $40,000 salary as manager with an S corporation. V. $1,200 of dividend income from Higher Mutual Fund. Assuming Marsha has sufficient at risk basis in each of the entities, what is Marsha's adjusted gross income? A) $35,700 B) $41,200 C) $38,200 D) $36,700

C) $38,200 Rationale The correct answer is "C." Option "I" - A loss from a limited partnership in which there is no material participation is governed under the passive activity loss rules. Since there is no other passive activity income to offset the loss, the loss is not currently deductible. Option "II" - The same passive activity loss rules apply, and therefore, the loss is not currently deductible. Option "III" - Because she is a material participant in managing the S corporation, the losses are deductible. Option "IV" - Wages are always included in AGI. Option "V" - Dividend income unless excluded is included in AGI. $40,000 (wages) minus $3,000 (S corp loss) plus $1,200 (dividends) = $38,200.

Ima Clipper, a well-known artist, donated one of her original bronze creations to a local charity, which auctioned the piece for $3,000. Ima totaled her costs as follows: Bronze = $425 Other materials = $150 Pro-rata overhead = $125 Furnace/casting fees = $200 Artistic contribution = $2,100 Assuming this is Ima's only charitable contribution and based on an annual income of $150,000, what is the maximum amount of charitable income tax deduction available to her? A) $3,000 B) $2,100 C) $775 D) $900

C) $775 Rationale The correct answer is "C." Only materials and expenses are deductible, not artistic contribution or time. No deduction is allowed for use of property; therefore, the pro-rata overhead would likely not be allowed.

Sabina Herz, a school teacher who is in a low marginal income tax bracket and is interested in investing in an educational business with some of her colleagues. Sabina says that she wants to keep the money in the business and doesn't want to pay taxes on income she never sees. Furthermore, Sabina wants to be assured the business would NOT be disrupted if one of her partners lost interest or encountered personal financial problems. What legal form of business makes the most sense, given Sabina's desires? A) A Limited Partnership. B) A General Partnership. C) A C Corporation. D) A Professional Corporation.

C) A C Corporation. Rationale The correct answer is "C." Options "A" and "B" are incorrect in that they would dissolve in the event of a 50% turnover in ownership in a 12-month period. Option "C" is correct because a C corporation generally retains income that is not passed to its owners through dividends. Option "D" is incorrect because it is structured as a C corporation for professional services. Not an educational business.

Alimony is not: A) Deductible for income taxed purposes by the payor spouse. B) Ordinary income to the recipient using the constructive receipt rule. C) Deductible paid to a third party if owed by recipient spouse without agreement. D) Recaptured in the third year if a front loaded property settlement.

C) Deductible paid to a third party if owed by recipient spouse without agreement. Rationale The correct answer is "C." Alimony is deductible by the payor spouse and taxable to the payee spouse if the agreement is dated prior to 12/31/18. There are no gift tax consequences related to qualifying alimony not deductible if paid to a third party without agreement by the recipient spouse.

Steven and Julie's children have the following for this year: - Brian, age 12, earns $2,500 in salary mowing lawns. - Courtney, age 19, earns $2,300 in dividends and capital gains. - Derek, age 16, earns $2,200 in dividends and interest. - Danny, age 10, earns $900 in dividends and interest. Whose income is subject to the tax at their trust and estate tax rate? A) Brian. B) Courtney and Derek. C) Derek. D) Brian, Courtney and Derek.

C) Derek. Rationale The correct answer is "C." In 2018, a person under the age of 19 or under the age of 24 and a full-time student at the end of the tax year, would pay tax at their tax trust and tax rate if they had more than $2,100 in unearned income. Thus, $100 of Derek's income is taxed at the trust and estate tax rate. Danny avoids this as $900 is under the standard deduction. Brian's income is earned and the kiddie tax applies only to unearned income and Courtney is 19. If the question stated that Courtney was a full-time college student then "B" would be the correct answer but you can't assume those facts.

Which of the following are preference items or adjustments for purposes of the individual alternative minimum tax? I. Qualified private-activity municipal bond interest. II. The excess of percentage depletion over the property's adjusted basis. III. Investment interest in excess of net investment income. IV. Qualified housing interest. A) I only. B) II and IV only. C) I and II only. D) II and III only.

C) I and II only. Rationale The correct answer is "C." By definition, investment interest expense in excess of net investment income and qualified housing interest are not preference items or adjustments for purposes of the alternative minimum tax.

Which of the following credentials permit an individual to appear before the IRS on behalf of a client? I. An enrolled agent. II. A Certified Public Accountant. III. An attorney IV. A CERTIFIED FINANCIAL PLANNER™. A) II and III only. B) I and II only. C) I, II and III only. D) I, III and IV only.

C) I, II and III only. Rationale The correct answer is "C." Only an attorney, enrolled agent, certified public accountant and the taxpayer can represent the taxpayer before the IRS. Certified Financial Planners, unless they are also an enrolled agent, CPA or attorney may NOT represent a third party taxpayer.

Sources of "substantial authority" available for tax research include: I. Internal Revenue Code. II. Congressional Committee Reports (Blue Book). III. Treasury Regulations. IV. Private Letter Rulings. A) I and II only. B) I, II, and III only. C) I, II, III and IV. D) I, III and IV only.

C) I, II, III and IV. Rationale The correct answer is "C." Substantial authority is official words and rulings which can be relied on to support a tax opinion or position. All of these can be relied on by someone.

Which of the following are characteristics of a C corporation? I. The number of shareholders is unlimited. II. The bankruptcy of a major shareholder has NO effect on the business form. III. Shareholder liability is limited. IV. The capital structure is dependent upon the resources of the shareholders. A) I and II only. B) III and IV only. C) I, II, and III only. D) I, II, III and IV.

C) I, II, and III only. Rationale The correct answer is "C." Options "II" and "III" are accurate. The number of shareholders is limited in an S corporation, but not in a C corporation. Capital may be raised by the sale of stock and by borrowing in capital markets.

How would the realization requirement influence an investor's decision to purchase stocks expected to appreciate in value but not paying dividends versus stocks paying dividends but not expected to appreciate in value? I. Dividends are recognized income in the year they are received. II. Appreciation in value is taxed in the year in which it occurs. III. Stock dividends are not recognized until the stock is sold. IV. Realization is an economic concept and recognition is a tax concept. A) I and IV only. B) II and III only. C) I, III, and IV only. D) I, II, and III only.

C) I, III, and IV only. Rationale The correct answer is "C." Dividends paid in cash are recognized as income in the year the dividends were paid. A dividend paid in shares of stock is not taxed upon receipt and will appreciate in value until the stock is sold. A cash based taxpayer recognizes income when received either actually or constructively. Realization is an economic concept, recognition is a tax concept.

The classifications of income are: I. Active Income. II. Earned Income. III. Unearned Income. IV. Portfolio Income. V. Passive Income. A) II, III, IV and V only. B) I, II and III only. C) I, IV and V only. D) All of the above.

C) I, IV and V only. Rationale The correct answer is "C." The classifications of income are active, passive and portfolio. Earned income is a subset of active income while unearned income may be either a passive or portfolio income.

Which of the following personal income tax planning techniques are used to defer (postpone) taxation? I. Itemizing deductions. II. Contributing to an individual retirement account. III. Using the child care credit. IV. Owning cash value life insurance. A) I and II only. B) II only. C) II and IV only. D) III and IV only.

C) II and IV only. Rationale The correct answer is "C." IRAs and cash value insurance both involve deferral techniques. The other options are avoidance or elimination techniques.

Which of the following is not generally one of the four main categories of itemized deductions? A) State income or sales tax, and property taxes. B) Charitable contributions. C) Personal casualty losses. D) Qualified personal residence mortgage.

C) Personal casualty losses. Rationale The correct answer is "C." The statement is incorrect since you can no longer deduct personal casualty losses.

What is the effective income tax rate? A) The highest marginal rate paid by a taxpayer based on income. B) The highest rate that is paid by an individual. C) The average rate a taxpayer pays based on taxable income. D) The tax rate considered for tax planning purposes.

C) The average rate a taxpayer pays based on taxable income. Rationale The correct answer is "C." The effective tax rate is the average tax an individual pays. The rate is determined by dividing the tax liability by the taxable income. The tax rate for planning purposes may be the effective or the highest marginal rate depending on the issue.

Under the accrual method of accounting, the taxpayer (buyer) recognizes expenses when: A) The goods leave the seller's warehouse. B) The seller accepts the buyer's purchase order. C) The buyer receives the seller's invoice. D) The buyer first uses the goods.

C) The buyer receives the seller's invoice. Rationale The correct answer is "C." The accrual accounting method recognizes expenses when the legal liability to pay arises. This usually occurs when the invoice is received.

Under the Last In First Out (LIFO) inventory system: A) The last goods purchased are the first goods sold. B) LIFO means that the oldest goods will remain in inventory until sold. C) The cost of goods is assigned the most current inventory costs. D) LIFO insures that the newest goods are sold.

C) The cost of goods is assigned the most current inventory costs. Rationale The correct answer is "C." The LIFO method is concerned with movement of costs through inventory, not goods. The cost of the last units purchased will be the first costs to be transferred to cost of goods sold when the goods are sold.

Explain the reason for the inclusion amount with respect to leased passenger automobiles. A) The inclusion amount estimates the difference between business and personal use. B) The inclusion amount is the result of the formula that determines whether it is best to lease or to buy an automobile. C) The inclusion amount is designed to help to level out the lease expense vs. the depreciation expense. D) The inclusion amount is the commuting miles that are included as part of the standard deduction on a leased auto, which cannot be deducted on an owned auto.

C) The inclusion amount is designed to help to level out the lease expense vs. the depreciation expense. Rationale The correct answer is "C." The inclusion amount is designed to help to smooth the lease expense vs. the depreciation expense. The lease is front loaded.

Janice, who is single, had gross income of $38,000, and incurred the following expenses: Charitable contributions = $2,500 Taxes and interest on home = $9,000 Legal fees incurred in a tax dispute = $1,000 Medical expenses = $4,000 Penalty on early withdrawal of savings = $200 Her AGI is: A) $38,000 B) $21,300 C) $29,000 D) $37,800

D) $37,800 Rationale The correct answer is "D." All but penalty for early withdrawal of savings are itemized deductions. The question asks to determine the AGI. Therefore, the AGI is $38,000 less $200 in penalties which equals $37,800.

On January 1st of this year, Linda sold a piece of land she had had for years to George. Linda's basis in the land was $75,000 and she sold it for $100,000. It was agreed that George would pay Linda $10,000 as a down payment and would make installment payments of $10,000 for the next 9 years plus 10% interest. His second payment was due and payable December 31 of this year. What is Linda's tax consequence of this transaction this year? A) $20,000 of ordinary income B) $20,000 of long term capital gain C) $2,500 of long term capital gain and $9,000 of ordinary income D) $5,000 of long term capital gain and $9,000 of ordinary income

D) $5,000 of long term capital gain and $9,000 of ordinary income Rationale The correct answer is "D." George is paying her $100,000. Her amount invested is $75,000. Therefore, over 10 years, her total profit will be $25,000 or $2,500 per year except for the down payment. There are two payments at the end of the year. An interest payment of 10% x $90,000*=$9,000 (ordinary income). The second payment, $10,000, consists of $2,500 capital gain and $7,500 of return of basis. *Recall the amount paid was $100,000 less a down payment of $10,000, so $90,000 was outstanding. Total: Down Payment $ 10,000= $ 2,500CG + $ 7,500RB $ - OI (no interest in this pmt_ Dec 31 Payment $ 10,000= $ 2,500CG + $ 7,500RB $ 9,000 OI (interest= 90k * 10%) TTL: 9k OI, 5k CG, 15k return of basis

Kevin was ordered to pay his ex-wife Janet $12,500 per year for support for 10 years. Kevin simply decided to pay her $50,000 the first and second year and then $25,000 in the third year, and Janet agreed. What will you tell him that the implications of this might be? A) There would be alimony recapture in the amount of $10,000. B) He has an alimony deduction of $50,000; $50,000; and $25,000 respectively. C) There would be alimony recapture in the amount of $15,000. D) There would be alimony recapture in the amount of $12,500.

D) There would be alimony recapture in the amount of $12,500. Rationale The correct answer is "D." P1 + P2 - 2P3 - $37,500 = Recapture, therefore $50,000 + $50,000 - 2($25,000) - $37,500 = $12,500 of alimony recapture.

Edward told his nephew that if the nephew would care for Edward in his old age, the nephew could have all of Edward's securities when he died. At the time of the promise, the securities had a fair market value of $50,000. The nephew took good care of Edward, whose will left the securities to the nephew. The fair market value of the securities at the time of Edward's death was $80,000. Edward could have gone to a nursing home and obtained the same services as provided by the nephew for $40,000. The nephew's gross income from the above is: A) $0; this is an inheritance. B) $40,000; this is earned income at the fair market value. C) $50,000; this is earned income as of the time of the promise. D) $80,000; this is earned income equal to the date of death value.

D) $80,000; this is earned income equal to the date of death value. Rationale The correct answer is "D." Because the agreement was to compensate Edward for his services, even though the transfer occurred following death, it is not a gift or bequest. It is compensation for services performed. Compensation of property has a value equal to its fair market value on the date of transfer. The fact that Edward died and a step up in basis would ordinarily occur is immaterial.

David is a wealthy attorney in the highest marginal income tax bracket. He is interested in purchasing a franchise in a fast-growing food chain with some of his associates. After reviewing the proposal, you have determined that apart from a substantial up-front investment, the business will NOT need to retain income and any income generated in subsequent years will be paid out to the investors. David wants to be assured the business would NOT be disrupted if one of his associates lost interest or encountered personal financial reversals. What form of business structure would make the most sense? A) A Limited Partnership. B) A General Partnership. C) A C Corporation. D) A S Corporation.

D) A S Corporation. Rationale The correct answer is "D." Options "A" and "B" are incorrect in that they would dissolve in the event of a 50% turnover in ownership in a 12-month period. Option "C" is incorrect because a C corporation generally retains income. Option "D" is correct as an S corporation holds no retained earnings since all profits or losses are passed through to its owners.

Which of the following distributions of IRC Section 1245 recapture property may result in the immediate recapture of some or all of previous depreciation deductions? A) A distribution by a partnership to one of its partners. B) A non-simultaneous like-kind exchange. C) A disposition at death. D) A sale for an interest-bearing note.

D) A sale for an interest-bearing note. Rationale The correct answer is "D." Section 1245 recapture is applied to the sale of depreciated assets. Option "A" is incorrect because the distribution is a property distribution and not a sale. Option "B" is incorrect because there is no "sale" as part of a like-kind exchange. Option "C" is incorrect because the property transferred at death is not classified as a sale. Option "D" is correct because it is a sale, regardless for cash, notes, either or both.

To what extent may the rental losses of an active participant be deducted against active and passive income? I. $25,000 of losses from rental property income may be deducted against ordinary income. II. The taxpayer must be considered "active" in that they participate in the general management and decision making of the property. III. The $25,000 is reduced $1 for every $2 over an AGI limit of $100,000. IV. When the AGI reaches $150,000, the deduction is lost and must be treated as regular passive income. A) I and II only. B) II and III only. C) II, III and IV only. D) All of the above.

D) All of the above. Rationale The correct answer is "D." $25,000 of losses from rental property income may be deducted against ordinary income. The taxpayer must be considered "active" in that they participate in the general management and decision making of the property. Also, the $25,000 is reduced $1 for every $2 over an AGI limit of $100,000. When the AGI reaches $150,000, the deduction is lost and must be treated as regular passive income.

Ivan invests in land and Grace invests in taxable bonds. The land appreciates by $5,000 each year, and the bonds earn interest of $5,000 each year. After holding the land and bonds for five years, Ivan and Grace sell them. There is a $25,000 realized gain on the sale of the land and no realized gain or loss on the sale of the bonds. Which of the following best describes the tax consequences to Ivan and Grace for the five years? I. Ivan will recognize a capital gain of $25,000 since appreciation (or the gain) on sale is not taxed until the asset is sold. II. Grace will have recognized the interest received over the years. III. Ivan will not have recognized gain over appreciation of unsold property. IV. Grace will not recognize a gain on sale since the bonds haven't appreciated. A) I only. B) II only. C) I, II and III only. D) All of the above.

D) All of the above. Rationale The correct answer is "D." Ivan will recognize a capital gain of $25,000 since appreciation or the gain on sale is not taxed until the asset is sold. Grace will not recognize a gain on sale since the bonds haven't appreciated. Grace will however have recognized the interest received over the years while Ivan will not have recognized gain over appreciation of unsold property.

The Qualified Dependent Test includes which of the following tests: I. Gross income. II. Support. III. Member of household or family. IV. Citizenship or residence. V. Joint return. A) I and II only. B) I, II, IV and V only. C) I, II and IV only. D) All of the above.

D) All of the above. Rationale The correct answer is "D." The five elements of the Qualified Dependent Test are: Gross income, support, member of household or family, citizenship or residence, and joint return.

Which of the following is a condition for receiving a dependent care credit? A) The taxpayer must provide over 1/2 cost of maintaining the household, which is also the principal residence of the child. B) The child must be a dependent. C) If married, both parents must work or go to school. D) All of the above.

D) All of the above. Rationale The correct answer is "D." These are all conditions for claiming the dependent care credit.

Can money paid for child support be structured in a divorce as to be deductible to the payor spouse? A) Yes, if the decree stipulates such. B) No, unless the taxpayer gets a letter ruling from the IRS. C) No, child support payments can never be made deductible. D) Yes, if the money to be considered as child support is included in deductible alimony

D) Yes, if the money to be considered as child support is included in deductible alimony Rationale The correct answer is "D." If an agreement is reached between former spouses where the decreed amount of alimony is increased to include child support, then the additional alimony would be taxable to the recipient and deductible to the payor. The additional money cannot be based on any contingency such as with the child reaching the age of majority or death.

Which of the following is/are more likely to result in the Internal Revenue Service challenging the reasonableness of compensation paid to a shareholder-employee? I. Compensation which is computed according to a contingent compensation agreement. II. Compensation paid by a company that has a history of no dividends. III. Compensation which is paid under a cost-of-living clause in an employment agreement. IV. Compensation paid by a company in a personal service business. V. Compensation paid by a company in a capital intensive business. A) IV only. B) I and III only. C) II and IV only. D) II and V only.

D) II and V only. Rationale The correct answer is "D." The question is directed to the issue of excessive compensation or compensation intended to forgo taxable income. Option "II" is suspect because dividends are not a deductible expense, and therefore, have no impact on lowering company income taxes. Option "V" is also suspect. The fact that the company is capital intensive indicates that it has a higher amount of revenue to expenses and a potentially high income tax burden. Both examples would be suspect in that the desire to reduce income tax requires the need to increase expenses, and therefore, gives rise to potential excessive or unreasonable compensation.

Larry, Mike and Clarence wish to start a manufacturing business. The business is expected to generate a large income which it will reinvest for many years. Larry has substantial assets which he plans to contribute to the business. Larry is also concerned about showing too much business income on his personal return. Which of the following structures would serve as an appropriate form of organization to satisfy Larry, Mike and Clarence's objective? I. A limited partnership with Larry as the limited partner. II. A business trust with all three as equal interests. III. An S corporation with all three as equal shareholders. IV. A C corporation with all three as shareholders. A) I, II and III only. B) I and IV only. C) II and IV only. D) IV only.

D) IV only. Rationale The correct answer is "D." Options "I," "II" and "III" are all pass through entities which will require the owner to report on their personal income tax return their pro rata share of income. Therefore, Options "I," "II" and "III" are all incorrect. The C corporation will not expose the shareholders to their share of undistributed corporate income.

Walter is a cash basis taxpayer. Which of the following items must be included in his adjusted gross income calculation for the current year. A) Earnings from Series EE bonds. B) Business income that was earned on Dec. 15 of the current year. The client mailed a check on Dec. 29 of the current year. The check arrived in Daniel's mailbox on Jan. 2 of the following year. C) A business sale to a customer made on Oct. 15. Daniel extended the payment due date, and the client has yet to pay the bill. D) Original Issue Discount on the bonds for the current year.

D) Original Issue Discount on the bonds for the current year. Rationale The correct answer is "D." Earnings on EE bonds can be delayed until redeemed or matured. Since he is a cash basis taxpayer, Walter does not have to include the business income until the year in which the check is received. The account receivable will not be included until paid. The original issue discount must be included in the current year's income regardless of receipt.

Assuming an asset is sold for a gain, when would Section 1250 ordinary income occur? A) Depreciable property is sold at a gain. B) Depreciable property is sold regardless of whether there is a gain or loss. C) Straight line depreciation is used on real property subject to ACRS. D) Real property subject to ACRS and accelerated depreciation was used.

D) Real property subject to ACRS and accelerated depreciation was used. Rationale The correct answer is "D." Section 1250 gain applies to the realized gain on real property where the accelerated method was used. The gain is the excess of accelerated over straight line (ACRS). Section 1250 gain is taxed as ordinary income. Under current law (MACRS), only straight line depreciation of real property is used.

Alberto Sanchez purchased a piece of equipment last year for his computer business. Which of the following depreciation methods would provide Alberto with the least depreciation during the period in question? A) MACRS. B) ACRS. C) Units of production. D) Straight line.

D) Straight line. Rationale The correct answer is "D." Although available as a tax depreciation method, straight line will provide the least depreciation expense for a given period. Option "A" provides the most depreciation, as well as one which will yield the greatest expense in the early portion of the asset's life. Option "B" is incorrect because ACRS application was discontinued in 1987. Option "C" is incorrect because it is not applicable to the type of equipment in question.

Under the accrual method of accounting, the taxpayer (seller) recognizes income when: A) The bill is received by the buyer. B) The goods are accepted by the buyer. C) The goods are loaded on the truck at the seller's facility. D) The seller writes and sends the invoice after sending the goods.

D) The seller writes and sends the invoice after sending the goods. Rationale The correct answer is "D." The accrual accounting method recognizes income when the taxpayer has a right to collect. This occurs usually after the completion of a job and in no case later than when the invoice is prepared and sent.

Freda purchased a stereo system for her son Wes, age 16. The stereo was placed in Wes' room and is used exclusively by him. Freda also purchased a new sports car in her own name, that was used 90% of the time by Wes. Which of the cost of these items may be considered as support in determining whether Freda may claim Wes as a dependent? A) Both the stereo and the car qualify as support because of the use test. B) Neither the stereo nor the car qualify as support because the car is Freda's and the stereo is diminimus. C) The stereo does not qualify for support but the car does because he uses it 90% of the time. D) The stereo qualifies for support, but the car does not even though it is de minimus.

D) The stereo qualifies for support, but the car does not even though it is de minimus. Rationale The correct answer is "D." The stereo system purchased and GIVEN to Wes qualifies as support. Because the car was not GIVEN to Wes (although he is allowed to use it) it will not be considered support. However, maintenance costs, such as gas and insurance that the taxpayer provides for his use of the car will qualify as support.

Which of the following is true of the substitute basis of a qualifying asset in a like-kind exchange? A) The substitute basis is the asset's fair market value increased by the gain realized but not recognized. B) The substitute basis is the asset's basis reduced by the gain realized but not recognized. C) The substitute basis is the asset's basis increased by the gain realized but not recognized. D) The substitute basis is the asset's fair market value reduced by the gain realized but not recognized.

D) The substitute basis is the asset's fair market value reduced by the gain realized but not recognized. Rationale The correct answer is "D." Substitute basis is the fair market value of an asset, reduced by gain realized, but not recognized.


संबंधित स्टडी सेट्स

Informations Systems and Data Analytic Ch. 1-2 Quiz

View Set

Social Impact of the Industrial Revolution Chapter 7 Section 3

View Set

Mental Health Test 3 [Monday 4/3/17]

View Set

Med Surg; Neuro/Muscular Questions

View Set

Pathology 3: Irreversible cell death - Apoptosis

View Set